You are on page 1of 92



150/-



      
      
 
 
      
LaExcellenceIAS CivilsPrepLaEx laexcellenceiasacademy
Civilsprep/NammaLaexBengaluru/ManaLaExManaKosam
RRP 2022 - ENVIRONMENT
UPSC PREVIOUS YEAR QUESTIONS

INDEX
Page No.
SL.No. Topic No. of Qns
From To

01. Environment and Ecology 46

02. Environmental Conservation 25

03. Contemporary Environment Issues 31

04. Environmental Management 53

05. Sustainable Development 19


     
RRP 2022 - Environment UPSC
Previous Year Questions
               
1.2. Ecosystem
1. Environment and 1.2.1. Concept of Ecosystem
Ecology 2. Which one of the following is the best
description of the term ‘Ecosystem'?
[2015]
1.1. Ecology
a. A community of organisms interacting with
1.1.1. Principles of Ecology
one another
1. Which one of the following terms b. That part of the Earth which is inhabited by
describes not only the physical space living organisms
occupied by an organism, but also its c. A community of organisms together with the
functional role in the community of environment in which they live
organisms? [2013] d. The flora and fauna of a geographical area
a. Ecotone Answer: C
b. Ecological niche Explanation: Option a is incorrect as it is
c. Habitat missing the abiotic component of environment
d. Home range • Part of earth on which we live is biosphere
Answer: B hence option b also incorrect.
Ecological Niche refers to the unique functional • Option c is correct because ecosystem is
role and position of a species in its habitat or formed with an interaction between the
ecosystem based on the its interactions with the biotic (living organisms) and abiotic (Soil,
ecosystem which helps in its survival and climate etc)).
reproduction. Hence, option B is correct. • Flora and Fauna are the components of
Ecosystem which also includes human
Ecotone is a zone of junction or a transition area
beings and abiotic components. Therefore,
between two biomes (diverse ecosystems).
option D is incorrect.
Habitat is the place or environment where a • Educational Objective: Ecosystem means
plant or animal naturally or normally lives and community of organisms reacting with
grows. biotic and abiotic components.
• Form: Single statement based
Home range is the area which has resources
• Style: Which
required for the survival and reproduction of an
• Substance: Terminology
organism.

Educational objective: Ecological Niche refers


to physical space occupied and functional role of
a community of organism.

Form: Single answered Question

Style: Which

Substance: Terminology

www.laexias.com Page 1 https://elearn.laex.in


 
     
RRP 2022 - Environment UPSC
Previous Year Questions
               
1.3. Bio Diversity d. Tamil Nadu
1.3.1. Genetic Diversity Answer: A
3. Recently, our scientists have discovered Explanation: Root Bridge
a new and distinct species of banana • Living root bridges (also known as Jing
plant which attains a height of about 11 KiengJri) is a result of handmade tree
metres and has orange-coloured fruit shaping activity by weaving and
pulp. In which part of India has it been manipulating the aerial roots of living trees
discovered? [2016] by Meghalaya tribes to create suspension
a. Andaman Islands bridges to cross streams and rivers.
b. Anaimalai forests • They have been serving as connectors for
c. Maikala Hills generations in the Indian state
d. Tropical rain forests of northeast of Meghalaya.
• They have also become world-
Answer: A
famous tourist attractions. The two most
Explanation: Scientists of Botanical Survey of
popular tourist spots are- Riwai Root
India (BSI) have discovered a new species of
Bridge and Umshiang Double Decker
banana known as Musa Acuminate from a
Bridge.
remote tropical Krishna Nalah rain forest on the
• Educational objective: In Meghalayathe
Little Andaman Islands.
local people train the roots of living trees
It is about 11 metres high, whereas as the usual
into robust bridges across the streams.
banana species is about three to four meters
• Form: Single answered based
high. The fruit pulp is orange in colour,
• Style: Description based question
distinctive from the white and yellow color of
• Substance: Current affair-based questions
regular bananas. Hence (a) is the correct
answer. 5. Other than resistance to pests, what are
the prospects for which genetically
Educational objective: Scientists of Botanical
engineered plants have been created?
Survey of India (BSI) have discovered a new
[2012]
species of banana in the Little Andaman Islands.
Form: Single answered based 1. To enable them to withstand drought
Style: Description based question 2. To increase the nutritive value of the
Substance: Species based question produce
3. To enable them to grow and do
4. In a particular region in India, the local
photosynthesis in spaceships and space
people train the roots of living trees into
stations
robust bridges across the streams. As the
4. To increase their shelf life
time passes, these bridges become
Select the correct answer using the codes
stronger. These unique 'living root
given below
bridges' are found in [2015]
a. 1 and 2 only
a. Meghalaya b. 3 and 4 only
b. Himachal Pradesh c. 1, 2 and 4 only
c. Jharkhand d. 1, 2, 3 and 4
www.laexias.com Page 2 https://elearn.laex.in
 
     
RRP 2022 - Environment UPSC
Previous Year Questions
               
Answer: c Taxol is used as cancer medication. It will slow
Explanation: Plants that have new genes down the growth of cancer cells and their spread
inserted into them are called transgenic or in the body.
genetically engineered / modified plants Taxol can also
Prospects: Educational objective: The Taxus tree naturally
Enhanced taste and quality. found in the Himalayas is listed in the Red Data
Reduced maturation time. Book.
Increased nutrients, yields, and stress tolerance. Form: Three statement Based question
Improved resistance to disease, pests, and Style: Consider based question
herbicides. Substance: Species based question
Educational objective: Genetically engineered 1.3.2. Species Diversity
plants have been created to enable them to 1.3.2.1. Flora Diversity (Plant)
withstand drought and to enable them to grow 7. Which of the following are detrivores?
and do photosynthesis in spaceships and space (2021)
stations.
1. Earthworms
Form: Four statements based
2. Jellyfish
Style: Description based question
3. Millipedes
Substance: Species based question
4. Seahorses
6. Consider the following statements: [2010] 5. Woodlice
1) The Taxus tree naturally found in the Select the correct answer using the code
Himalayas given below
2) The Taxus tree is listed in the Red Data (a) 1, 2 and 4 only
Book (b) 2, 3, 4 and 4 only
3) A drug called "Taxol" is obtained from Taxus (c) 1, 3 and 5 only
tree is effective against Parkinson's disease (d) 1, 2, 3, 4 and 5
Which of the statements given above is/are Answer: c
correct? Explanation:
a. 1 only Detritivores are heterotrophs that obtain their
b. 1 and 2 only nutrition by feeding on detritus i.e. dead or
c. 2 and 3 only decaying plants or animals.
d. 1,2 and 3 Statements 1, 3 and 5 are correct.
Earthworms, Millipedes and Woodlice are
Answer: b
detritivores. Detritivores include microorganisms
Explanation: Taxuswallichiana, the Himalayan
such as bacteria and fungi; invertebrate insects
yew, is a species of yew, native to the Himalaya
such as mites, beetles, butterflies and flies;
and parts of south-east Asia. The species has a
mollusks such as slugs and snails; or soil-
variety of uses in traditional medicine. It is
dwelling earthworms, millipedes and woodlice.
currently classified as endangered by the IUCN.
Jellyfish is carnivorous and Sea horse is also
primarily a carnivorous animal.

www.laexias.com Page 3 https://elearn.laex.in


 
     
RRP 2022 - Environment UPSC
Previous Year Questions
               
Educational objective: Earthworms, Millipedes 9. In nature, which of the following is/are
and Woodlice are detrivores. most likely to be found surviving on a
Form: List based question surface without soil? (2021)
Style: Which based questions 1. Fern
Substance: Species based question 2. Lichen
8. What is blue carbon? (2021) 3. Moss
(a) Carbon captured by oceans and coastal 4. Mushroom
ecosystems Select the correct answer using the code
(b) Carbon sequestered in forest biomass and given below
agricultural soils (a) 1 and 4 only
(c) Carbon contained in petroleum and natural (b) 2 only
gas (c) 2 and 3
(d) Carbon present in the atmosphere (d) 1, 3 and 4
Answer: a Answer: c
Explanation: Explanation:
Blue Carbon refers to coastal, aquatic and 1) Lichens have specific requirements for their
marine carbon sinks held by vegetation, marine habitats. These requirements are: water, air,
organisms and sediments. The coastal nutrients, light, and substrates. Every lichen
ecosystems of mangroves, tidal marshes and lives on top of something else. The surface of
seagrass meadows contain large stores of carbon that “something else” is called a substrate. Just
deposited by vegetation and various natural about anything that holds still long enough for a
processes over centuries. lichen to attach to and grow is a suitable
These ecosystems sequester and store more substrate. Trees, rocks, soil, houses,
carbon – often referred to as ‘blue carbon’ – per tombstones, cars, old farm equipment and more
unit area than terrestrial forests. The ability of can be substrates. The most common natural
these vegetated ecosystems to remove carbon substrates are trees and rocks.
dioxide (CO2) from the atmosphere makes them 2) Mosses are non-flowering plants which
significant net carbon sinks, and they are now produce spores and have stems and leaves,
being recognised for their role in mitigating but don’t have true roots. So, without roots,
climate change. some moss suck nutrients up through the
Educational objective: Carbon captured by rhizoids and others draw in moisture and
oceans and coastal ecosystems is known as blue minerals from rain and the water around
carbon. them through their highly absorbent surfaces.
Form: Single statement based It often grows from trees, absorbing water and
Style: What based questions nutrients from the atmosphere, but with proper
Substance: Terminology based questions care it can also grow indoors.
3) Fern is a member of a group of vascular
plants (plants with xylem and phloem). They
grow in soils. There are four particular types of
habitats that ferns are found in: moist,
www.laexias.com Page 4 https://elearn.laex.in
 
     
RRP 2022 - Environment UPSC
Previous Year Questions
               
shady forests; crevices in rock faces, especially lives in a symbiotic relation with algae as a
when sheltered from the full sun; acid wetlands lichen.
including bogs and swamps; and tropical trees. Statement 3 is correct. Protozoa also forms
4) A mushroom or toadstool is the fleshy, symbiotic relations with other organisms.
spore-bearing fruiting body of a fungus, Symbiosis in protozoa mostly represents:
typically produced above ground, on soil, or on • a close mutualistic association between a
its food source. protozoan and unicellular symbiont (like
Educational objective: Lichen and Moss most bacteria, cyanobacteria or/and unicellular
likely to be found surviving on a surface without algae) or
soil. • protozoans and a multicellular organism
Form: List based question (ruminants, lower termites, wood-eating
Style: Which based questions cockroaches, plants)
Substance: Species based question
Educational objective: Cnidarians, Fungi and
10. Which of the following have species that Protozoa can establish a symbiotic relationship
can establish a symbiotic relationship with other organisms
with other organisms? (2021) Form: List based question
1. Cnidarians Style: Which based questions
2. Fungi Substance: Species based question
3. Protozoa 11. Which one of the following is a filter
Select the correct answer using the code feeder? (2021)
given below: (a) Catfish
(a) 1 and 2 only (b) Octopus
(b) 2 and 3 only (c) Oyster
(c) 1 and 3 only (d) Pelican
(d) 1, 2 and 3 Answer: c
Answer: d Explanation:
Explanation: Option c is correct. Oysters are natural filters.
Some organisms live together and share both They capture the particles of size as small as 4
shelter and nutrients. This relationship is called micrometers on their gills. Their gills act as filter
symbiosis. which is self-cleaning.
Statement 1 is correct. Cnidaria is an Generally, water get filtered by something which
invertebrate phylum which often involves in a is static so that water can easily pass through
symbiotic relation with the unicellular them and get filtered. Pelicans, Catfish and
dinoflagellate algae, called zooxanthellae. Octopus are large organisms. Oysters are small
Statement 2 is correct. Certain fungi live inside organisms, found cemented to rocks or other
the roots of plants. The plants provide nutrients hard substrates.
to the fungus and, in return, the fungus Educational objective: Oysters are natural
provides water and certain nutrients. Fungi also filters of water
Form: Single answered

www.laexias.com Page 5 https://elearn.laex.in


 
     
RRP 2022 - Environment UPSC
Previous Year Questions
               
Style: Which based questions Form: List based question
Substance: Species based question
Style: Consider based question
12. Consider the following animals (2021)
Substance: Species based question
1. Hedgehog
2. Marmot 13. Which one of the following groups of
3. Pangolin plants was domesticated in the ‘New
World’ and introduced into the ‘Old
To reduce the chance of being captured by
World’? [2019]
predators, which of the above organisms rolls
up/roll up and protects/protect its/their a. Tobacco, cocoa and rubber
vulnerable parts? b. Tobacoo, cotton and rubber
c. Cotton, coffee and sugarcane
(a) 1 and 2
d. Rubber, coffee and wheat
(b) 2 only
(c) 3 only Answer: a
(d) 1 and 3 Explanation: Tobacco, cocoa and rubber were
originally domesticated or cultivated in the ‘New
Answer: d
World’ (America) and introduced into the ‘Old
Explanation World’ (Asia and Africa). Cotton and Wheat are
being cultivated in India since very ancient
Option 1 is correct. Hedgehogs are small
times. People of Mehrgarh (Baluchistan,
nocturnal spiny mammals. When they are
Pakistan) cultivated Cotton during Neolithic age.
frightened, or annoyed, hedgehogs will roll into a
Wheat was cultivated by people of Harappan
ball so that a predator will feel the full brunt of
civilisation and Vedic Aryans
its sharp spines and will then leave the
Domestication of plants means adapting the wild
hedgehog alone.
plants for domestic purposes like food, clothing
Option 2 is incorrect. Marmots are relatively etc…
large ground squirrels with 15 species living in Educational objective: Tobacco, cocoa and
Asia, Europe, and North America. They are rubber plants was domesticated in the ‘New
the heaviest members of the squirrel family. It World’ and introduced into the ‘Old World’.
does not roll up to protect itself. Form: Single statement based
Style: Which based questions
Option 3 is correct. Pangolins are actually
Substance: Species based question
mammals. They are the only mammals wholly-
covered in scales and they use those scales to 14. Recently, there was a growing awareness
protect themselves from predators in the wild. in our country about the importance of
Under threat, a pangolin will immediately curl Himalayan nettle (Girardiniadiversifolia)
into a tight ball and will use their sharp-scaled because it is found to be a sustainable
tails to defend themselves. source of [2019]
(a) Anti-malarial drug
Educational objective: Hedgehog and Marmot
(b) Biodiesel
roll up to protect their vulnerable parts.
(c) Pulp for paper industry
www.laexias.com Page 6 https://elearn.laex.in
 
     
RRP 2022 - Environment UPSC
Previous Year Questions
               
(d) Textile fibre Educational objective: Red Sanders/Red
Answer: d Sandalwood (a tree) is endemic (i.e., native) to
the Eastern Ghats in South India.
Explanation: Girardiniadiversifolia (Himalayan
nettle), a fibre-yielding plant is naturally Form: Two statements based
available in the foothills of Himalayas like Nepal, Style: Reference Based
India (Uttarakhand, Himachal Pradesh and Substance: Species based question
J&K). It is the longest fiber and is finer, stronger 16. Government of India encourages the
and more elastic than linen. Khar community in cultivation of 'sea buckthorn'. What is
Nepal, produces fabrics from Himalayan nettle. the importance of this plant? [2012]
The fabric and the things made from it are sold
1) It helps in controlling soil erosion and in
in local as well as national and international
preventing desertification.
markets as high-end products.
2) It is a rich source of bio-diesel.
Educational objective: Himalayan nettle 3) It has nutritional value and is well-adapted
(Girardiniadiversifolia) is found to be a to live in cold areas of high altitudes.
sustainable source of Textile fibre. 4) Its timber is of great commercial value.
Form: Single answered based
Which of the statements given above is/are
Style: Description based question correct?
Substance: Species based question
(a) 1 only
15. With reference to 'Red Sanders', (b) 2, 3 and 4 only
sometimes seen in the news, consider (c) 1 and 3 only
the following statements: [2016] (d) 1, 2, 3 and 4
1) It is a tree species found in a part of South Answer: c
India. Explanation: Buckthorn
2) It is one of the most important trees in the It has multi-purpose medicinal and nutritional
tropical rain forest areas of South India. properties, and also helps in soil conservation
Which of the statements given above is/are and nitrogen fixation.
correct? Hardy, drought-resistant and tolerant to extreme
(a) 1 only temperatures from – 43º C to + 40º C, the plant
(b) 2 only has an extensive root system which can fix
(c) Both 1 and 2 atmospheric nitrogen, making it ideal for
(d) Neither 1 nor 2 controlling soil erosion and preventing
desertification.
Answer: A
National mission on buckthorn in is a part of
Explanation: Red Sanders/Red Sandalwood (a
Sub-Mission on Cold Desert Ecosystems under
tree) is endemic (i.e., native) to the Eastern
the Green India Mission — which is a part of the
Ghats in South India. Therefore, statement 1
National Action Plan on Climate Change.
is correct.
Educational objective: 'sea buckthorn' it helps
It occurs in the forest formation which is
in controlling soil erosion and in preventing
classified as “Southern Tropical Dry Deciduous
desertification and has nutritional value and is
Forests”. Therefore, statement 2 is incorrect.
www.laexias.com Page 7 https://elearn.laex.in
 
     
RRP 2022 - Environment UPSC
Previous Year Questions
               
well-adapted to live in cold areas of high 3) One-horned rhinoceros is naturally found in
altitudes. India only.
Form: Four statements.
Style: What based questions. Which of the following statement given above
Substance: Species based question. is/are correct?
17. Some species of plants are insectivorous. a. 1 only
Why? [2010] b. 2 only
(a) Their growth in shady and dark places does c. 1 and 3 only
not allow them to undertake sufficient d. 1,2 and 3
photosynthesis and thus they depend on Answer: a
insects for nutrition Explanation: Asiatic lions are slightly smaller
(b) They are adapted to grow in nitrogen than African lions. At present Gir National Park
deficient soils and thus depend on insects and Wildlife Sanctuary is the only natural abode
for sufficient nitrogenous nutrition of the Asiatic lion. Double-hump camels are
(c) They cannot synthesize certain vitamins found naturally in Tibet, Mountain regions of
themselves and depend on the insects China, Mangolia, Pakistan and also in mountain
digested by them regions of Afghanistan. One horned rhino is
(d) They have remained in that particular stage found in India, Pakistan,Nepal and lower
of evolution as living fossils, a link between reached of Bhutan.
autotrophs and heterotrophs Educational objective: Asiatic lion is naturally
Answer: b found in India only.
Explanation: Insectivorous plants prey insects Form: Three statement Based question
because most of the insectivorous plants grow in Style: Consider based question
those area where there is deceit of nitrogen in Substance: Species based question
nutrients so they catch insect to full its need of 19. Consider the following pairs [2019]
nutrients.
Wildlife Naturally found in
Educational objective: Insectivorous plants are
1) Blue- finned Mahseer : Cauvery River
adapted to grow in nitrogen deficient soils and
2) Irrawaddy dolphin : Chambal River
thus depend on insects for sufficient
3) Rusty-spotted cat : Eastern Ghats
nitrogenous nutrition.
Which of the pairs given above are correctly
Form: Single statement based
matched?
Style: Why based questions
Substance: Species based question (a) 1 and 2 only
1.3.2.2. Fauna Diversity (Wild Life) (b) 2 and 3 only
1.3.2.2.1. Distribution (c) 1 and 3 only
(d) 1,2 and 3
18. Consider the following statements: [2019]
Answer: c
1) Asiatic lion is naturally found in India only.
Explanation:
2) Double-humped camel is naturally found in
Blue- finned Mahseer : Cauvery River
India only.

www.laexias.com Page 8 https://elearn.laex.in


 
     
RRP 2022 - Environment UPSC
Previous Year Questions
               
Irrawaddy dolphin : Chilka lake (not 21. In which of the following regions of India
Chambal River) are you most Likely to come across the
Rusty-spotted cat : Eastern Ghats 'Great Indian Hornbill' in its natural
Therefore, by eliminating ‘2’, option ‘C’ is habitat? [2016]
correct.
Educational objective: Blue- finned Mahseer
(a) Sand deserts of northwest India
naturally found in Cauvery River, Rusty-spotted
(b) Higher Himalayas of Jammu and Kashmir
cat naturally found in Eastern Ghats and
(c) Salt marshes of western Gujarat
Irrawaddy dolphin found in Chilka lake.
(d) Western Ghats
Form: Matching based question.
Style: Consider based question. Answer: d
Substance: Species based question. Explanation: Great Hornbill or Great Indian
Hornbill is one of the larger members of the
20. If you want to see gharials in their
hornbill family. It is found in Nepal, South East
natural habitat, which one of the
Asia, Western Ghats and forests of Himalayas.
following is the best place to visit? [2017]
Under IUCN Red List, great Indian Hornbill
(a) Bhitarkanika Mangroves comes under Near Threatened and listed in
(b) Chambal River Appendix 1 of CITES.
(c) Pulicat Lake Educational objective: Great Indian Hornbill is
(d) Deepor Beel found in Nepal, South East Asia, Western Ghats
Answer: b and forests of Himalayas.
Explanation: Gharials once inhabited all the Form: Single answered based
major river systems of the Indian Subcontinent, Style: Which based questions
from the Irrawaddy River in the east to the Indus Substance: Habitat/Protected areas-based
River in the west. But now, their distribution is questions
limited to only 2% of their former range. 22. If you walk through countryside, you are
In India, Gharials can be found in Girwa River, likely to see some birds stalking
Chambal River, Ken River, Son River, Mahanadi alongside the cattle to seize the insects
River, Ramganga River disturbed by their movement through
gharial is one of three crocodiles that are native grasses. Which of the following is/are
to India, the other two are the mugger crocodile such bird/birds? [2014]
and the saltwater crocodile.
1) Painted Stork
Educational objective: Gharials best natural
2) Common Myna
habitate is Chambal river.
3) Black-necked Crane
Form: Single answered based.
Style: Which based questions. Select the correct answer using the code
Substance: Habitat/Protected areas-based given below.
questions. (a) 1 and 2 (b) 2 only
(c) 2 and 3 (d) 3 only

www.laexias.com Page 9 https://elearn.laex.in


 
     
RRP 2022 - Environment UPSC
Previous Year Questions
               
Answer: b They are the rarest and most threatened
Explanation: Common Myna are birds stalking primates of old-world monkeys.
alongside the cattle to seize the insects Educational objective: Lion-tailed macaque
disturbed by their movement through grasses. found in Tamil Nadu, Kerala and Karnataka.
The common myna is readily identified by the Form: List based question
brown body, black hooded head and the bare Style: Which based questions
yellow patch behind the eye. Substance: Habitat/Protected areas-based
Painted Stork feeds mainly on small fish in questions
shallow wetlands
24. Consider the following: [2013]
Black-necked Crane feed on waste grains in
1) Star tortoise
agricultural valleys
2) Monitor lizard
Educational objective: Common Myna likely to
3) Pygmy hog
see some birds stalking alongside the cattle to
4) Spider monkey
seize the insects.
Form: List based question Which of the above are naturally found in
Style: Description based question India?
Substance: Species based question a. 1, 2 and 3 only
23. In which of the following States is lion- b. 2 and 3 only
tailed macaque found in its natural c. 1 and 4 only
habitat? [2013] d. 1,2,3 and 4
Answer: a
1) Tamil Nadu
Explanation: Star tortoise is found in India in
2) Kerala
the dry and scrub forests. Pygmy Hog is an
3) Karnataka
endangered species found in Assam. Only 150
4) Andhra Pradesh
animals are left. Monitor Lizard is found in
Select the correct answer using the codes India, Sri Lanka and Pakistan. Spider Monkey
given below is the inhabitant of tropical forests of Central
a. 1,2 and 3 only and South America.
b. 2 only Educational objective: Star tortoise, Monitor
c. 1,3 and 4 only lizard and Pygmy hog are naturally found in
d. 1,2,3 and 4 India.
Answer: a Form: List based question
Explanation: lion-tailed macaque mainly Style: Consider based question
diurnal arboreal, it prefers the upper canopy of Substance: Species based question
primary tropical evergreen rainforest but may 25. Consider the following: [2012]
also be found in monsoon forest in hilly country 1) Black-necked crane
and in disturbed forest. 2) Cheetah
It can be found in Karnataka, Tamil Nadu and 3) Flying squirrel
Kerala in the western ghat region. 4) Snow leopard
Unlike other macaques, it avoids humans.

www.laexias.com Page 10 https://elearn.laex.in


 
     
RRP 2022 - Environment UPSC
Previous Year Questions
               
Which of the above are naturally found in Answer: a
India? Explanation:
a. 1, 2 and 3 only As per the Elephant census report, a greater
b. 1, 3 and 4 only number of Elephants are present in South India.
c. 2 and 4 only Among the south Indian states Karnataka leads
d. 1, 2, 3 and 4 the list followed by Kerala. In the recent times,
Answer: b Kerala witnessed a marginal decline in the
Explanation: Species naturally found in India elephants.

1) Black-necked crane Elephants have the longest gestation period of


2) Flying squirrel all mammals, carrying their young for 18 to 22
3) Snow leopard months before giving birth. Long developmental
Cheetah, the fastest animal inhabits in Africa periods are common among highly intelligent
and parts of Middle East animals. So, option A is the answer.
Educational objective: Black-necked crane,
Female elephants can go for calving till the age
Flying squirrel and Snow leopard are naturally
of 70 also. So, third statement is wrong.
found in India.
Educational Objective: The leader of an Indian
Form: List based question
elephant group is a female and maximum
Style: Consider based question
gestation period can be 22 months.
Substance: Species based question
Form: Four statements
1.3.2.2.2. Scientific name Style: Reference Based
1.3.2.2.3. Characteristics Substance: Species based question
26. With reference to Indian elephants, 27) With reference to India’s biodiversity,
consider the following statements: [2020] Ceylon frogmouth, Coppersmith barbet,
1. The leader of an elephant group is a female Gray-chinned minivet and White-
2. The maximum gestation period can be 22 throated redstart are [2020]
months. a) Birds
3. An elephant can normally go on calving till b) Primates
the age of 40 years only. c) Reptiles
4. Among the States in India, the highest d) Amphibians
elephant population is in kerala.
Answers: a
Which of the statement given above is/ are Explanation:
correct? The Sri Lanka frogmouth, Sri Lankan frogmouth
a) 1 and 2 only or Ceylon frogmouth is a small frogmouth (The
b) 2 and 4 only frogmouths are a group of nocturnal birds)
c) 3 only found in the Western Ghats of south India and
d) 1,3 and 4 only Sri Lanka.
The coppersmith barbet, also called crimson-
breasted barbet and coppersmith, is an Asian
barbet with crimson forehead and throat, known
www.laexias.com Page 11 https://elearn.laex.in
 
     
RRP 2022 - Environment UPSC
Previous Year Questions
               
for its metronomic call that sounds similar to a these camels swim to islands near the shore in
coppersmith striking metal with a hammer. It is search of food.
a resident bird in the Indian subcontinent and They are domesticated. In the recent times, the
parts of Southeast Asia. camel breeders have been raising concerns over
The grey-chinned minivet is a species of bird in their dwindling numbers. So, statement ‘3’ is
the family Campephagidae. It is found from the wrong as it says it lives only in wild.
Himalayas to China, Taiwan and Southeast Asia. Educational objective: 'Kharai camel' is
The white-throated redstart is a species of bird capable of swimming up to three kilometre’s in
in the family Muscicapidae. It is found in Nepal, seawater.It survives by grazing on mangroves.
Bhutan, central China and far northern areas of Form: Three statement Based question
Myanmar and Northeast India. Its natural Style: What based questions
habitat is temperate forests. Hence option (a) is Substance: Species based question
the correct answer.
29. With reference to 'dugong', a mammal
Educational Objective: Ceylon frogmouth,
found in India which of the following
Coppersmith barbet, Gray-chinned minivet and
statements is/is correct? [2015]
White- throated redstart are birds.
Form: Single answered based 1) It is a herbivorous marine animal.
Style: Reference Based 2) It is found along the entire coast of India.
Substance: Species based question 3) It is given legal protection under Schedule I
of the Wildlife (Protection) Act; 1 972.
28. What is/are unique about 'Kharai camel'
a breed found in India? [2016] Select the correct answer using the code
given below.
1) It is capable of swimming up to three
kilometre’s in seawater. a. 1 and 2
2) It survives by grazing on mangroves. b. 2 only
3) It lives in the wild and cannot be c. 1 and 3
domesticated. d. 3 only

Select the correct answer using the code Answer: c


given below. Explanation: The dugong is a medium-
sized marine mammal. It is one of four living
a. 1 and 2 only
species of the order Sirenia, which also includes
b. 3 only
three species of manatees. It is the only living
c. 1 and 3 only
representative of the once-diverse family
d. 1, 2 and 3
Dugongidae; its closest modern relative, Steller's
Answer: A sea cow (Hydrodamalisgigas), was hunted
Explanation: Kharai camels have a special to extinction in the 18th century. The dugong is
ability to swim in seawater and feed on saline the only strictly herbivorous marine mammal.
plants and mangroves, which is how they get It is found only in the coastal areas between
their name, Kharai ('salty' in Gujarati). They are India and Sri Lanka. Therefore, Statement 2 is
also known as dariyataru (meaning sea- not correct.
swimmer). Due to their unique diet,

www.laexias.com Page 12 https://elearn.laex.in


 
     
RRP 2022 - Environment UPSC
Previous Year Questions
               
Due to threats like death and habitat Which of the above is / are mammal
destruction caused by human interference in the /mammals?
form of fishing, illegal hunting, destruction of a. 1 only
coral reefs, shallow water trawling etc…. b. 1 and 3 only
Dugong’s were listed under Schedule 1 of c. 2 and 3 only
Wildlife protection Act, 1972 which confers d. 1,2 and 3
highest level of legal protection.
Key: b
Educational objective: The dugong is a
Explanation: Sea cow is a mammal. It is a
medium-sized marine mammal, it is listed under
herbivore and it grows up to 9 meters. It looks
Schedule 1 of Wildlife protection Act, 1972.
like a large seal. Sea horse is a fish not a
Form: Three statement Based question
mammal. Sea lion is a mammal. It is a
Style: Reference Based
carnivore. It is also known as eared seal.
Substance: Species based question
Educational objective: Sea horse is a fish not a
30. Which one of the following is the mammal.
national aquatic animal of India? [2015] Form: List based question
a. Saltwater crocodile Style: Consider based question
b. Olive ridley turtle Substance: Species based question
c. Gangetic dolphin 32. Consider the following kinds of
d. Gharial organisms [2012]
Answer: c 1) Bat
Explanation: The Gangetic dolphins have been 2) Bee
declared as the National Aquatic Animal of India. 3) Bird
River Dolphin is the National Aquatic Animal of
Which of the above is/are pollinating agent /
India. The Ministry of Environment and
agents?
Forests notified the Ganges River Dolphin as
the National Aquatic Animal on 18th May a. 1 and 2 only
2010. This mammal is also said to represent the b. 2 only
purity of the holy Ganga as it can only survive in c. 1 and 3 only
pure and fresh water. d. 1, 2 and 3
Educational objective: The Gangetic dolphins Answer: d
have been declared as the National Aquatic Explanation: Insects, birds, and bats are biotic
Animal of India. pollinating agents. Water, wind and plants are
Form: Single answered based abiotic pollinating agents. So, all the above
Style: Which based questions options are pollinating agents.
Substance: Species based question Educational objective: Bat, Bee and Bird are
31. Consider the following Animals [2013] pollinating agents.
Form: List based question
1) Sea cow
Style: Consider based question
2) Sea horse
Substance: Species based question
3) Sea lion

www.laexias.com Page 13 https://elearn.laex.in


 
     
RRP 2022 - Environment UPSC
Previous Year Questions
               
33. King Cobra is the only snake that makes Select the correct answer using the code
its own nest. Why does it make its nest? given below
[2010] a. 1 only
a. It is a snake-eater and the nest helps attract b. 2 and 3 only
other snakes c. 1 and 3 only
b. It is a viviparous snake and needs a nest to d. 1, 2 and 3
give birth to its offspring Answer: D
c. It is an oviparous snake and lays its eggs in Explanation: The equatorial, hot, wet climate is
the nest and guards the nest until they are
found between 5° and 10° north and south of
hatched
the equator. Its greatest extent is found in the
d. It is a large, cold blooded animal and needs
lowlands of the Amazon, the Congo, Malaysia
a nest to hibernate in the cold season
and the East Indies.
Answer: B • Equatorial rainforests are the typical
Explanation: The king cobra is the only snake vegetation type found in this region.
that builds a nest using dry leave litter, starting • The region is characterized by broad-leaf
from late March to late May. Most nests are trees with a tall and dense canopy.
located at the base of trees, are up to 55 cm • The canopy is so dense that very little
(22 in) high in the center and 140 cm (55 in) sunlight reaches the surface of equatorial
wide at the base. They consist of several layers forests.
and have mostly one chamber, into which the • Because of abundant sunlight and
female lays eggs. precipitation, the growing season exists all
Educational objective: King Cobra is a around the year. There is no distinct
viviparous snake and needs a nest to give birth drying season where the trees shed leaves.
to its offspring. Shedding and blooming of leaves and fruits
Form: Single statement based occur throughout the year.
Style: Why based questions • The vegetation exists in layers with the
Substance: Species based question bottom-most layer comprising of decaying
organic matter, which is slow to decompose
1.3.3. Ecosystem Diversity due to low sunlight. A very dense layer of
canopy exists with trees growing as tall as
34. Which of the following is/are unique
90 feet - these layers most of the wildlife of
characteristic / characteristics of
these forests. Above this lies the emergent
equatorial forests? [2013]
layer of crowns of a few trees which
1) Presence of tall closely set trees with crowns protrude out of the dense canopy. This
forming a continuous canopy. layer receives maximum sunlight and also
2) Coexistence of a large number of species. has to endure strong winds.
3) Presence of numerous varieties of epiphytes. • On the thick trunks of many of the trees,
there is a layer of climbers known as the
lianas. Epiphytes are small plants which

www.laexias.com Page 14 https://elearn.laex.in


 
     
RRP 2022 - Environment UPSC
Previous Year Questions
               
grow on the trunks and branches of these Style: Consider based question
broad-leaf trees. Substance: Habitat/Protected areas-based
• The rainforests of Amazon are known as questions.
Selvas (forests with a dense canopy)
• The trees of the hardwood variety. 1.3.4. Hot Spots of Bio Diversity
Prominent hardwood varieties include -
36. The Himalayan Range is very rich in
Mahogany, Ebony, dyewoods etc.
species diversity. Which one among the
• Apart from the dense hardwood forests,
following is the most appropriate reason
mangrove vegetation is also found in the
for this phenomenon? [2011]
swamps and coastal regions.
Educational Objective: To know about the a. It has a high rainfall that supports
natural vegetation around the world. luxuriant vegetative growth.
b. It is a confluence of different bio
35. Consider the following statements: [2011]
geographical zones.
1) Biodiversity is normally greater in the lower c. Exotic and invasive species have not been
latitudes as compared to the higher introduced in this region.
latitudes. d. It has less human interference.
2) Along the mountain gradients, biodiversity
Answer: B
is normally greater in the lower altitudes as
Explanation: The Himalayan Range is very rich
compared to the higher altitudes.
in species diversity because it is a confluence of
Which of the statements given above is/are different bio-geographical zones.
correct? The Himalaya Hotspot is home to the world's
a. 1 only highest mountains, including Mt. Everest. The
b. 2 only mountains rise abruptly, resulting in a diversity
c. Both 1 and 2 of ecosystems that range from alluvial
d. Neither 1 nor 2 grasslands and subtropical broadleaf forests to
alpine meadows above the tree line. Vascular
Answer: C
plants have even been recorded at more than
Explanation: Biodiversity is higher in the low
6,000 m. The hotspot is home to important
latitudes as compared to the higher latitudes as
populations of numerous large birds and
the habitat tolerance level is very narrow at
mammals, including vultures, tigers,
greater latitude. Mountains have high range of
elephants, rhinos and wild water buffalo.
species at the low latitude as they can support
Educational objective: The Himalayan Range is
larger number of species due to climatic
a confluence of different bio geographical zones
condition. As higher we go, the colder will be the
so it is very rich in species diversity.
atmosphere and less biodiversity.
Educational objective: Biodiversity is higher in Form: Single statement based
the low latitudes as compared to the higher
Style: Description based question
latitudes as the habitat tolerance level is very
narrow at greater latitude. Substance: Habitat / Protected areas-based
Form: Two statements questions

www.laexias.com Page 15 https://elearn.laex.in


 
     
RRP 2022 - Environment UPSC
Previous Year Questions
               
37. Three of the following criteria have Substance: Habitat/Protected areas-based
contributed to the recognition of Western questions
Ghats-Sri Lanka and Indo-Burma regions 1.3.5. Importance of Bio Diversity
as hotspots of biodiversity: [2011]
1.3.5.1. Ecosystem Services
1) Species richness
2) Vegetation density 38. Biodiversity forms the basis for human
3) Endemism existence in the following ways: [2011]
4) Ethno-botanical importance 1) Soil formation
5) Threat perception 2) Prevention of soil erosion
6) Adaptation of flora and fauna to warm and 3) Recycling of waste
humid conditions 4) Pollination of crops
Which three of the above are correct criteria Select the correct answer using the codes
in this context? given below:
a. 1, 2 and 6 a. 1,2 and 3 only
b. 2, 4 and 6 b. 2, 3 and 4 only
c. 1, 3 and 5 c. 1 and 4 only
d. 3, 4 and 6 d. 1, 2, 3 and 4
Answer: C Answer: D
Explanation: To qualify as a hotspot, a region Explanation: Biodiversity plays a major role in
must meet two strict criteria: maintaining the ecological balance of the
It must contain at least 1,500 species of ecosystem. It refers to the number of different
vascular plants (> 0.5 percent of the world’s species belonging to a particular region. In
total) as endemics, and it has to have lost at biodiversity, each individual species has a major
least 70 percent of its original habitat. So we role to play in the ecosystem.
choose Species richness as well as Endemism. Ecological Role of Biodiversity
Along with this Threat perception is necessary to Apart from providing ecological balance to the
take, because it makes the base of this concept. environment, each individual species of
Adaptation of flora is an arbitrary option, Ethno- biodiversity has a major function to play in the
botanical importance does nothing with the ecosystem. They play a major role in the
Biodiversity Hotspot selection criteria, vegetation production and decomposition of organic wastes,
Density is also discarded. fixing atmospheric gases and regulation of water
Educational objective: Species richness, and nutrients throughout the ecosystem. The
Endemism and Threat perception have stability of the ecosystem increases with the
contributed to the recognition of Western Ghats- diversity of the species.
Sri Lanka and Indo-Burma regions as hotspots Economical Role of Biodiversity
of biodiversity. Biodiversity acts as a source of energy and has a
major role in providing raw materials for
Form: List based question
industrial products such as oils, lubricants,
Style: Description based question perfumes, dyes, paper, waxes, rubber, etc.

www.laexias.com Page 16 https://elearn.laex.in


 
     
RRP 2022 - Environment UPSC
Previous Year Questions
               
Importance of plant species for various d. Mankind's over-exploitation/misuse of
medicinal use has been known since ages. natural resources, fragmentation/loss of
According to reports, more than 70 % of the natural habitats, destruction of ecosystems,
anti-cancer drugs are derived from plants in the pollution and global climate change.
tropical rainforests. Answer: D
Scientific Role of Biodiversity Explanation: The Holocene extinction,
Each species of the ecosystem contributes to otherwise referred to as the sixth mass
providing enough evidence as to how life evolved extinction or Anthropocene extinction, is an
on this planet and the role of each species in ongoing extinction event of species during the
maintaining the sustainability of the ecosystem. present Holocene epoch (with the more recent
Educational objective: Soil formation, time sometimes called Anthropocene) as a result
Prevention of soil erosion, Recycling of waste and of human activity.
Pollination of crops forms the basis for human • More than 30% of animals with a backbone
existence in the following ways. — fish, birds, amphibians, reptiles and
Form: List based question mammals — are declining in both range and
Style: Description based question population.
Substance: Cause and effect-based questions • The mammal species that were monitored
have lost at least a third of their original
1.3.6. Threats to Bio Diversity habitat. 40% of them — including rhinos,
39. The term "sixth mass extinction/sixth orangutans, gorillas and many big cats —
extinction" is often mentioned in the are surviving on 20% or less of the land they
news in the context of the discussion of once roamed. The loss of biodiversity has
[2018] recently accelerated.
• Several species of mammals that were
a. Widespread monoculture practices in
relatively safe one or two decades ago are
agriculture and Large-scale commercial
now endangered, including cheetahs, lions
farming with indiscriminate use of
and giraffes. There are as few as 20,000 lions
chemicals in many parts of the world that
left in the wild, less than 7,000 cheetahs,
may result in the loss of good native
500 to 1,000 giant pandas, and about 250
ecosystems.
Sumatran rhinoceros.
b. Fears of a possible collision of a meteorite
• Tropical regions have seen the highest
with the Earth in the near future in the
number of declining species. In South and
manner it happened 65 million years ago
Southeast Asia, large-bodied species of
that caused the mass extinction of many
mammals have lost more than four-fifths of
species including those of dinosaurs.
their historical ranges.
c. Large scale cultivation of genetically
Factors behind the loss?
modified crops in many parts of the world
The main drivers of wildlife decline are habitat
and promoting their cultivation in other
loss, overconsumption, pollution, invasive
parts of the world which may cause the
species, disease, as well as poaching in the case
disappearance of good native crop plants
of tigers, elephants, rhinos and other large
and the loss of food biodiversity.
www.laexias.com Page 17 https://elearn.laex.in
 
     
RRP 2022 - Environment UPSC
Previous Year Questions
               
animals prized for their body parts. Climate Educational objective: Fall in the population of
change is poised to become a major threat in the species of butterflies could affect Pollination of
coming decades. some plants and reduce population of some
Educational objective: "sixth mass species of wasps, spiders and birds.
extinction/sixth extinction" refers to Mankind's Form: Three statement Based question
over-exploitation/misuse of natural resources, Style: Description based question
fragmentation/loss of natural habitats, Substance: Species based question
destruction of ecosystems, pollution and global 41. Other than poaching, what are the
climate change. possible reasons for the decline in the
Form: Single statement based population of Ganges River Dolphins?
Style: Reference Based [2014]
Substance: Current affair-based questions 1) Construction of dams and barrages on
rivers
40. Due to some reasons, if there is a huge
2) Increase in the population of crocodiles in
fall in the population of species of
rivers
butterflies, what could be its likely
3) Getting trapped in fishing nets accidentally
consequence/consequences? [2017]
4) Use of synthetic fertilizers and other
1) Pollination of some plants could be agricultural chemicals in crop-fields in the
adversely affected. vicinity of rivers.
2) There could be a drastic increase in the Select the correct answer using the code
fungal infections of some cultivated plants. given below.
3) It could lead to a fall in the population of a. 1 and 2 only
some species of wasps, spiders and birds. b. 2 and 3 only
Select the correct answer using the code c. 1, 3 and 4 only
given below: d. 1, 2, 3 and 4
Answer: C
a. 1 only
Explanation: Ganges River Dolphin is placed
b. 2 and 3 only
under “Endangered Category” in the IUCN
c. 1 and 3 only
Red List. It lives in one of the world’s most
d. 1,2 and 3
densely populated areas, and is threatened by
Answer: C removal of river water and siltation arising from
Explanation: Butterflies pollinate some wild deforestation, pollution and entanglement in
flowers. Since some species of wasps, spiders fishing nets. They have been poached over for
and birds are predators of butterflies, decline of their oil. The habitat degradation due to
the latter could lead to a fall in the declining ow, heavy siltation and construction of
population of the predators. barrages causing physical barrier for this
Butterflies keep the population of fungus in migratory species is also one of the reasons
check by eating them. But here, the word behind decline of their numbers.
‘drastic’ is extreme. Fall in butterflies cannot Increase in crocodile population does not
lead to drastic increase of fungal infections. threaten these dolphins

www.laexias.com Page 18 https://elearn.laex.in


 
     
RRP 2022 - Environment UPSC
Previous Year Questions
               
Educational Objective: To know about the Their Range includes central and northern India
important species, and their unique features. and southern Nepal
And species classification by international Educational objective: Gharial is Critically
organizations and Acts. Endangered, Leatherback sea turtle is listed in
42. Consider the following fauna of India: Vulnerable and swamp deer or Barasingha listed
[2013] in Vulnerable (VU) category.
1. Gharial Form: List based question
2. Leatherback turtle Style: Consider based question
3. Swamp deer Substance: Species based question

Which of the above is/are endangered? 43. Which of the following can be threats to
the biodiversity of a geographical area?
a. 1 and 2 only
[2012]
b. 3 only
c. 1,2 and 3 1) Global warming
d. None of the above 2) Fragmentation of habitat
3) Invasion of alien species
Answer: D
4) Promotion of vegetarianism
Explanation: Gharial
Critically Endangered— IUCN Red List Select the correct answer using the codes
Gharials once inhabited all the major river given below:
systems of the Indian Subcontinent, from the a. 1, 2 and 3 only
Irrawaddy River in the east to the Indus River in b. 2 and 3 only
the west. Their distribution is now limited to c. 1 and 4 only
only 2% of their former range. It now includes d. 1, 2, 3 and 4
Girwa River, Chambal River, Ken River, Son Key: A
River, Mahanadi River etc., Explanation: Some of the major threats
Leatherback sea turtle The natural habitat of the ecosystem plays a
It is listed in Vulnerable category of IUCN and major role in maintaining the ecological balance.
also in CITES Appendix I species. Several trees are cut down every year for the
It is the largest of all living turtles. They can be construction of industries, highways, settlement
found primarily in the open ocean and nesting and so on to fulfil the human demands. As a
areas are in the tropics. result, the species become the target to
Swamp deer or Barasingha predation and eventually die.
It is listed in Vulnerable (VU) category – IUCN. It Hunting of the wild animals for
is the State animal of Madhya Pradesh and Uttar commercialisation of their products has been a
Pradesh. major reason for the loss of biodiversity. Since
They prefer tall grasslands and open habitats. the year 2013, more than 90 rhinos were killed
They found in swampland and a variety of forest by the poachers for their horns and as per the
types ranging from dry to moist deciduous to records of 2016, 9 Indian Rhinos have been
evergreen. They also found in grassy floodplains, killed in Kaziranga National Park of Assam.
wooded areas, and found near water bodies

www.laexias.com Page 19 https://elearn.laex.in


 
     
RRP 2022 - Environment UPSC
Previous Year Questions
               
The exploitation of the medicinal plants for 44. Vultures which used to be very common
several laboratory purposes has resulted in the in Indian countryside some years ago are
extinction of these species. Also, several animals rarely seen nowadays. This is attributed
are sacrificed for the purpose of various research to [2012]
in science and medicine. a. the destruction of their nesting sites by new
Natural calamities like floods, earthquakes, invasive species
forest fires also lead to the loss of biodiversity. b. a drug used by cattle owners for treating
Air pollution has a major role in the loss of their diseased cattle
biodiversity. Rapid cutting down of the trees has c. scarcity of food available to them
resulted in the increase of carbon dioxide in the d. a widespread, persistent and fatal disease
atmosphere leading to climate change. As a among them
result, there has been an increase in the land
Answer: B
and ocean temperature leaving an inimical
Explanation: The major reason behind the
impact on species.
vulture population getting nearly wiped out was
the drug Diclofenac. It was found in the carcass
of cattle on which the vultures feed.
Diclofenac is used on cattle, to treat
inflammation, fever or wound related pains
When Animals are treated with Diclofenac, this
drug keeps accumulating in their body. When
these animals die, Vultures eat them leading to
kidney failure and death of vulture.
This led to huge decline of vulture population.
So, government has banned use of diclofenac.
Educational objective: to know about the
important species, and their unique features
Promotion of vegetarianism will not result in loss
of biodiversity. By shifting vegetarian food, 45. Which one of the following groups of
animals will not be killed and forests will not be animals belongs to the category of
converted into agriculture fields. endangered species? [2012]
Educational objective: Global warming, a. Great Indian Bustard, Musk Deer, Red
Fragmentation of habitat and Invasion of alien Panda and Asiatic Wild Ass
species can be threats to the biodiversity of a b. Kashmir Stag, Cheetal, Blue Bull and Great
geographical area. Indian Bustard
Form: List based question c. Snow Leopard, Swamp Deer, Rhesus
Style: Which based questions Monkey and Saras (Crane)
Substance: Cause and effect-based questions d. Lion-tailed Macaque, Blue Bull, Hanuman
Langur and Cheetal.

www.laexias.com Page 20 https://elearn.laex.in


 
     
RRP 2022 - Environment UPSC
Previous Year Questions
               
Answer: A
Explanation: 2. Environmental
Great Indian Bustard, Musk deer, Asiatic Wild Conservation
Ass and Red Panda are categorised by IUCN and
Wildlife Institute of India (WII) as Endangered 2.1. Biodiversity Conservation
species
2.1.1. In-situ
Educational objective: Great Indian Bustard,
2.1.1.1. National Parks
Musk Deer, Red Panda and Asiatic Wild Ass
belongs to the category of endangered species. 1. With reference to India’s Desert National
Form: Single statement based Park, which of the following statements
Style: Which based questions are correct? [2020]
Substance: Species based question
1. It is spread over two districts.
46. A sandy and saline area is the natural 2. There is no human habitation inside the
habitat of an Indian animal species. The Park.
animal has no predators in that area but 3. It is one of the natural habitats of Great
its existence is threatened due to the Indian Bustard.
destruction of its habitat. Which one of
Select the correct answer using the code
the following could be that animal? [2011]
given below
a. Indian wild buffalo
a) 1 and 2 only
b. Indian wild ass
b) 2 und 3 only
c. Indian wild boar
c) 1 and 3 only
d. Indian gazelle
d) 1, 2 and 3
Answer: B
Answer: C
Explanation:
Indian wild ass is a threatened species Explanation:
(according to IUCN) found only in the Rann of
India’s desert National park is located in the
Kutch which is saline and sandy.
state of Rajasthan and it is the only species in
100 years ago, Indian wild ass lived all over the
India. In any national park, the human
dry regions of north western India and western
habitations that are existing allowed to continue,
Pakistan including Jaisalmer, Bikaner, Sind and
so desert National park is not an exception. It is
Baluchistan.
one of the natural habitats of the Great Indian
Educational Objective: Indian wild ass is a
Bustard which is a critically endangered species.
threatened species (according to IUCN) found
By using elimination techniques. Hence, Option
only in the Rann of Kutch which is saline and
c is the correct answer
sandy.
Form: Single answered based Educational objective: India’s Desert National
Style: Description based question Park is spread over two districts and it is one of
Substance: Species based question the natural habitats of Great Indian Bustard.

Form: Three statement Based question

www.laexias.com Page 21 https://elearn.laex.in


 
     
RRP 2022 - Environment UPSC
Previous Year Questions
               
Style: Reference Based 3. Which one of the following National
Parks lies completely in the temperate
Substance: Habitat / Protected areas-based
alpine zone? [2019]
questions
a. Manas National Park
2. Which one of the following protected b. Namdapha National Park
areas is well-known for the conservation c. Neora Valley National Park
of a sub-species of the Indian swamp deer d. Valley of Flowers National Park
(Barasinga) that thrives well in hard
Answer: D
ground and is exclusively graminivorous?
[2020] Explanation: The Valley of Flowers national
a) Kanha National Park park located in Uttarakhand at approximately
b) Manas National Park 3255m above sea level. It is known for its
c) Mudumalai Wildlife Sanctuary meadows of endemic alpine flowers and the
d) Tal Chhapar Wildlife Sanctuary variety of flora.

Answer: A The valley has three sub-alpine between 3,200m


and 3,500m which is the limit for trees, lower
Explanation:
alpine between 3,500m and 3,700m, and higher
• Barasingha is the one of the rarest species of alpine above 3,700m.
deer family spread across central and
The rich diversity of species reflects the valley's
northern India only in small congregations.
location within a transition zone between the
The deer is native to India and Nepal.
Zanskar and Great Himalayas ranges to the
Barasingha are also known as swamp deer
north and south, respectively, and between the
due to their nature to survive close to river
Eastern Himalaya and Western Himalaya flora.
valleys or swampy regions.
Educational Objective Valley of Flowers
• Kanha National Park is located in Madhya
National Park lies completely in the temperate
Pradesh and a renowned place for tigers. But
alpine zone.
the park’s major achievement is to save
nearly extinct barasingha (swamps deers) Form: Single answered based

• Educational objective: Kanha National Park Style: Which based questions


well-known for the conservation of a sub-
Substance: Habitat/Protected areas-based
species of the Indian swamp deer
questions
(Barasinga).
4. Recently there was a proposal to
• Form: Single answered based
translocate some of the lions from their
• Style: Which based questions natural habitat in Gujarat to which one
of the following sites? [2017]
• Substance: Habitat/Protected areas-based
questions a. Corbett National Park
b. Kuno Palpur Wildlife Sanctuary

www.laexias.com Page 22 https://elearn.laex.in


 
     
RRP 2022 - Environment UPSC
Previous Year Questions
               
c. Mudumalai Wildlife Sanctuary Educational objective: Keibullamjao national
d. Sariska National Park park in Manipur is the only floating national
Answer: B park in the world.

Explanation: In order to conserve the single Form: Single answered based


isolated population of Asiatic Lions, the Ministry Style: Which based questions
of Environment and Forests had initiated habitat
feasibility study through the Wildlife Institute of Substance: Habitat / Protected areas-based
India, which inter alia has identified KunoPalpur questions
Wildlife Sanctuary in Madhya Pradesh as an 6. Which one of the following National
alternate home for the Gir Lions and for the Parks has a climate that varies from
release of wild lions from Gujarat. While State tropical to subtropical, temperate and
Government of Madhya Pradesh is agreeable for arctic? [2015]
translocation of Gir Asiatic Lions in KunoPalpur,
a. Khangchendzonga National Park
State Government of Gujarat has so far not
b. Nandadevi National Park
agreed to part with Gir Asiatic Lions.
c. Neora Valley National Park
Educational objective: There was a proposal to d. Namdapha National Park
translocate some of the lions from Kuno Palpur
Answer: D
Wildlife Sanctuary.
Explanation: Namdapha National Park is part
Form: Single answered based
of Eastern Himalaya biodiversity hotspot in
Style: Which based questions Arunachal Pradesh.

Substance: Habitat / Protected areas-based In this region, climate varies from tropical to
questions subtropical, temperate and arctic.

5. Which of the following National Parks is Tropical and subtropical climate is found in the
unique in being a swamp with floating southern region and arctic type is found in
vegetation that supports a rich northern part of the park.
biodiversity? [2015]
Educational objective: Namdapha National
a. Bhitarkanika National Park Park has a climate that varies from tropical to
b. KeibulLamjao National Park subtropical, temperate and arctic.
c. Keoladeo Ghana National Park
Form: Single answered based
d. Sultanput National Park
Answer: B Style: Which based questions

Explanation: Keibul Lamjao National Park, Substance: Habitat / Protected areas-based


Manipur is the only floating national park in the questions
world. It’s a swamp, provides refuge to Eld's deer
or Manipuri Deer, listed as an endangered
species by IUCN.

www.laexias.com Page 23 https://elearn.laex.in


 
     
RRP 2022 - Environment UPSC
Previous Year Questions
               
7. Consider the following pairs: [2013] 8. Consider the following pairs: [2013]
National Park River flowing through the Park Protected area Well-known for

1. Corbett National Park Ganga 1. Bhiterkanika, Orissa Salt Water


2. Kaziranga National Park Manas Crocodile
3. Silent Valley National Park Kaveri 2. Desert National Park, Great Indian
Which of the above pairs is/are correctly RajasthanBustard
matched? 3. Eravikulam, Kerala Hoolak Gibbon
Which of the pairs given above is/are
a. 1 and 2
correctly matched?
b. 3 only
c. 1 and 3 a. 1 only
d. None b. 1 and 2 only
Answer: D c. 2 only
d. 1, 2 and 3
Explanation: Through Corbett National Park,
Answer: B
Ramganga flows (not Ganga) which is a tributary
of Ganges. Through Silent Valley National Park, Explanation: Bhiterkanika, Orissa is a
river Bhavani flows which is a tributary of protected area for salt water crocodile, where
Kaveri. breeding is the main purpose of that protected
area.
Kaziranga and Manas are two different national
parks • Great Indian Bustard is protected in desert
area of Rajasthan.
Manas river (tributary of Brahmaputra) flows
through Manas National Park • The Eravikulam National Park was
established to protect the Niligiri tahir
Whereas river Brahmaputra flows through
species.
Kaziranga
• Educational objective: The Eravikulam
So, all the above are not correctly matched.
National Park was established to protect the
Educational objective: Ramganga flows Niligiri tahir species.
through Corbett National Park, and river
• Form: Matching based question
Bhavani flows through Silent Valley National
Park. • Style: Consider based question

Form: Matching based question • Substance: Habitat/Protected areas-based


questions.
Style: Consider based question

Substance: Habitat / Protected areas-based


questions.

www.laexias.com Page 24 https://elearn.laex.in


 
     
RRP 2022 - Environment UPSC
Previous Year Questions
               
9. In which one among the following Select the correct answer using the code
categories of protected areas in India are given below
local people not allowed to collect and
a) 1 and 2 only
use the biomass? [2012]
b) 2 and 3 only
a. Biosphere Reserves c) 3 and 4 only
b. National Parks d) 1 and 4 only
c. Wetlands declared under Ramsar
Answer: A
Convention
d. Wildlife Sanctuaries Explanation:
Answer: B Hint: Musk deer is a species found in hilly
Explanation: In national parks, local people are habitat with reasonable altitudes. Kishanpur
not allowed to collect and use the biomass and Manas are located on plains.
available in the areas. • Askot Musk Deer Sanctuary is located in
National Park is an area that protects and Uttarakhand state of India. This sanctuary
conserves fauna, flora, geological formations has been set up primarily with the object of
etc…. conserving the musk deer and its habitat.

Here hunting, killing, grazing or capturing of • Gangotri National Park is home to the snow
fauna or collection of flora or deprivation of leopard. Musk deer is also part of this
wildlife etc…… are prohibited. habitat

Educational objective: In national parks, local • The Kishanpur Wildlife Sanctuary is a part of
people are not allowed to collect and use the the Dudhwa Tiger Reserve Uttar Pradesh,
biomass available in the areas. India. Musk deer is not found here. The
fauna of the Manas sanctuary include Indian
Form: Single answered based elephants, Indian rhinoceros, gaurs, Asian
Style: Which based questions water buffaloes, and barasingha but not
musk deer.
Substance: Habitat / Protected areas-based
questions • Educational Objective: Askot Wildlife
Sanctuary and Gangotri National Park most
2.1.1.2. Wild life sanctuaries
likely places to find the musk deer in its
10. Which of the following are the most natural habitat.
likely places to find the musk deer in its • Form: List based question
natural habitat? [2020]
• Style: Which based questions
1. Askot Wildlife Sanctuary
2. Gangotri National Park • Substance: Habitat/Protected areas-based
3. Kishanpur Wildlife Sanctuary questions
4. Manas National Park

www.laexias.com Page 25 https://elearn.laex.in


 
     
RRP 2022 - Environment UPSC
Previous Year Questions
               
11. In which one of the following States is Answer: A
Pakhui Wildlife Sanctuary located? [2018]
Explanation:
a. Arunachal Pradesh
b. Manipur Agasthyamalai Biosphere Reserve is located in
c. Meghalaya the Western Ghats.
d. Nagaland It is also a unique genetic reservoir of cultivated
Answer: A plants Ex: cardamom, jamune, nutmeg, pepper
and plantain.
Explanation: Pakhui Wildlife Sanctuary also
known as Pakhui Tiger Reserve is located at Three wildlife sanctuaries, Shendurney, Peppara
Arunachal Pradesh. and Neyyar of Kerala are located in the site, as
well as the Kalakad Mundanthurai Tiger reserve
It is a project Tiger reserve and consists of wide
of Tamil Nadu
variety of animals that stay in the forest without
human interventions. This Tiger Reserve is Educational objective: Neyyar, Peppara and
encircled with beautiful rivers in the three sides. Shendurney Wildlife Sanctuaries; and
KalakadMundanthurai Tiger Reserve are in
Educational objective: Pakhui Wildlife
Agasthyamala Biosphere Reserve.
Sanctuary located in Arunachal Pradesh.
Form: Single statement based
Form: Single answered based
Style: Which based questions
Style: Which based questions
Substance: Habitat/Protected areas-based
Substance: Habitat/Protected areas-based
questions
questions
13. The most important strategy for the
2.1.1.3. Biosphere Reserves
conservation of biodiversity together
12. Which of the following are in with traditional human life is the
Agasthyamala Biosphere Reserve? [2019] establishment of [2017]
a. Neyyar, Peppara and Shendurney Wildlife a. Biosphere reserves
Sanctuaries; and KalakadMundanthurai b. Botanical gardens
Tiger Reserve c. National parks
b. Mudumalai, Sathyamangalam and d. Wildlife sanctuaries
WayanadWildlife Sanctuaries; and Silent Answer: A
Valley National Park.
c. Kaundinya, GundlaBrahmeswaram and Explanation: As per Forest ministry guideline
Papikonda Wildlife Sanctuaries; and “Biosphere Reserves are special entities (sites)
Mukurthi National Park. for how human beings and nature can co-exist
d. Kawal and Sri Venkateswara Wildlife while respecting each other’s needs”. Biosphere
Sanctuaries; and Nagarjunasagar-Srisailam reserve- goal is to foster economic Development
Tiger Reserve which is ecologically sustainable. transition
zone, the outermost part of the Biosphere
www.laexias.com Page 26 https://elearn.laex.in
 
     
RRP 2022 - Environment UPSC
Previous Year Questions
               
Reserve, is an area of active Form: Single answered based
cooperation between the management and
Style: Which based questions
the local people, wherein activities like
settlements, cropping, forestry , recreation Substance: Habitat / Protected areas-based
and other economic that are in harmony with questions
the conservation goals
15. From the ecological point of view, which
Educational objective: Biosphere reserves are one of the following assumes importance
the most important strategy for the conservation in being a good link between the Eastern
of biodiversity together with traditional human Ghats and the Western Ghats? [2017]
life.
a. Sathyamangalam Tiger Reserve
Form: Single answered based b. Nallamala Forest
c. Nagarhole National Park
Style: Description based question d. Seshachalam Biosphere Reserve
Substance: Habitat / Protected areas-based Answer: A
questions
Explanation: Sathyamangalam forest range is
2.1.1.4. Species oriented projects a significant wildlife corridor in the Nilgiri
2.1.1.4.1.1. Project Tiger Biosphere Reserve between the Western Ghats
and the rest of the Eastern Ghats and a genetic
14. Among the following Tiger Reserves,
link between the four other protected areas
which one has the largest area under
which it adjoins, including the Billigiriranga
“Critical Tiger Habitat”? [2020]
Swamy Temple Wildlife Sanctuary, Sigur
a) Corbett Plateau, Mudumalai National Park and
b) Ranthambore Bandipur National Park.
c) Nagarjunsagar-Srisailam
d) Sunderbans
Answer: C

Explanation:

Nagarjunsagar-Srisailam Tiger Reserve (NSTR) is


the largest tiger reserve in India. The reserve
spreads over five districts, Nalgonda District,
Mahbubnagar district, Kurnool District,
Prakasam District and Guntur District Its core
area under the critical tiger habitat is also the
highest in India.
Educational objective: Sathyamangalam forest
Educational Objective: Nagarjunsagar-
range is good link between the Eastern Ghats
Srisailam has the largest area under “Critical
and the Western Ghats.
Tiger Habitat”

www.laexias.com Page 27 https://elearn.laex.in


 
     
RRP 2022 - Environment UPSC
Previous Year Questions
               
Form: Single answered based Wildlife Sanctuary is a National Park, UNESCO
Natural World Heritage site, a Project Tiger
Style: Description based question
Reserve, an Elephant Reserve and a Biosphere
Substance: Habitat/Protected areas-based Reserve in Assam. The Sundarban National Park
questions is a National Park, Tiger Reserve, and a
Biosphere Reserve in India. It is a part of the
16. The term 'M-STRIPES' is sometimes seen
Sundarbans on the Ganges Delta of India and
in the news in the context of [2017]
Bangladesh. Bhitarkanika National Park is a
a. Captive breeding of Wild Fauna national park located in the Kendrapara District
b. Maintenance of Tiger Reserves Odisha, which is not specifically for Tiger
c. Indigenous Satellite Navigation System reserve. It is home to saltwater crocodiles,
d. Security of National Highways. Indian Python, rhesus monkey, etc…..
Answer: B Educational objective: Bandipur, Manas and
Explanation: The android-based monitoring Sunderbans are declared Tiger Reserves.
software M-STrIPES will be used across all the Form: List based question
Tiger Reserves of the country.
Style: Consider based question
M-STrIPES = Monitoring System for Tigers
Intensive Protection and Ecological Status Substance: Habitat/Protected areas-based
questions
Educational objective: initiatives, and
2.1.1.5. Mixed categories
technology usage in the conservation of wildlife

17. Consider the following protected areas 18. Which of the following Protected Areas
[2012] are located in Cauvery basin? [2020]

1) Bandipur 1. Nagarhole National park


2) Bhitarkanika 2. Papikonda National park
3) Manas 3. Sathyamangalam Tiger Reserve
4) Sunderbans 4. Wayanad Wildlife sanctuary
Select the correct answer using the code
Which of the above are declared Tiger
given below:
Reserves?
a) 1 and 2 only
a. 1 and 2 only b) 3 and 4 only
b. 1, 3 and 4 only c) 1,3 and 4 only
c. 2, 3 and 4 only d) 1,2,3 and 4
d. 1, 2, 3 and 4 Answer: C
Answer: B
Explanation:
Explanation: Bandipur National Park, a tiger
Hint: Elimination technique, Papikonda is in
reserve is located in the south Indian state of
news, which is there in Godavari Basin of
Karnataka. Manas National Park or Manas
Andhra Pradesh.

www.laexias.com Page 28 https://elearn.laex.in


 
     
RRP 2022 - Environment UPSC
Previous Year Questions
               
Nagarhole National Park is in the Cauvery Basin. Educational objective: Dampa Tiger Reserve,
Sathyamangalam Tiger Reserve lies on the the largest wildlife sanctuary in Mizoram.
banks of the River Bhavani, a tributary of the Saramati peak is in Nagaland.
River Cauvery in the foothills of the Eastern
Form: Matching based question
Ghats. Kabini river (a tributary of Cauvery river)
flows through the Wayanad wildlife sanctuary. Style: Consider based question
Whereas Papikonda remains entirely inside the
Substance: Habitat / Protected areas-based
East and West Godavari districts near Godavari
questions.
basin after 2014 and after the construction of
Polavaram Dam. Hence, option 3 is correct 20. Consider the following pairs: [2013]
answer.
1) Nokrek Biosphere Reserve Garo Hills
Educational Objective: Nagarhole National 2) Logtak (Loktak) Lake Barail
park, Sathyamangalam Tiger Reserve and Range
Wayanad Wildlife sanctuary are located in 3) Namdapha National Park Dafla Hill
Cauvery basin. Which of the above pairs is/are correctly
Form: List based question matched?

Style: Which based questions a. 1 only


b. 2 and 3 only
Substance: Habitat / Protected areas-based c. 1,2 and 3
questions d. None

19. Consider the following pairs: [2014] Answer: A

1) Dampa Tiger Reserve Mizoram Explanation: Nokrek is the highest peak in


2) Gumti Wildlife Sanctuary Sikkim Garo Hills in Meghalaya
3) Saramati Peak Nagaland
Loktak lake is the largest fresh water lake
Which of the above pairs is/are correctly
famous for Phumdis floating over it in Manipur.
matched?
Barail range is in Assam
a. 1 only
Though Namdapha National Park and Dafla Hill
b. 2 and 3 only
both are in Arunachal Pradesh, the two are
c. 1 and 3 only
separate entities.
d. 1,2 and 3
Educational objective: Loktak lake floating over
Answer: C
it in Manipur.
Explanation: Dampa Tiger Reserve, the largest
Form: Matching based question
wildlife sanctuary in Mizoram. Saramati peak
is in Nagaland. It is located near Tuensang town Style: Consider based question
with a height of 3,826 m. Gumti Wildlife
Substance: Habitat / Protected areas-based
Sanctuary is famous wildlife reserve in Tripura.
questions

www.laexias.com Page 29 https://elearn.laex.in


 
     
RRP 2022 - Environment UPSC
Previous Year Questions
               
21. Consider the following statements: [2010] 2.1.2. Ex-situ
2.1.2.1. Botanical gardens, Zoo’s
1) The boundaries of a National Park are
defined by legislation 22. Which one of the following is not a site
2) A Biosphere Reserve is declared to conserve for in-situ method of conservation of
a few specific species of flora and fauna flora? [2011]
3) In a Wildlife Sanctuary, limited biotic
a. Biosphere Reserves
interference is permitted
b. Botanical Garden
Which of the statements given above is/are c. National Park
correct? d. Wildlife Sanctuary
a. 1 only Answer: B
b. 2 and 3 only
Explanation:
c. 1 and 3 only
d. 1,2 and 3
Answer: C

Explanation: A biosphere reserve conserves an


ecosystem, not just few specific species of plants
and animals.

The difference between a national park and a


sanctuary is that no human activity is allowed
inside a national park, while limited activities Educational objective: Botanical Garden is not
are permitted within the sanctuary. a site for in-situ method of conservation of flora.
Educational objective: to know about the Form: Single statement based
National parks, wildlife sanctuaries and Style: Which based questions
biosphere reserves classification, climatic Substance: Habitat / Protected areas-based
characteristics of the regions and the location. questions
World Network of Biosphere Reserves 2.2. Conservation of
Form: Three statement Based question Soil and Land
Style: Consider based question 23. In India, the use of carbofuran, methyl
parathion, prorate and triazophos is
Substance: Habitat / Protected areas-based
viewed with apprehension. These
questions.
Chemicals are used as [2019]
a. Pesticides in agriculture
b. Preservatives in processed foods
c. Fruit-ripening agents
d. Moisturizing agents in cosmetics

www.laexias.com Page 30 https://elearn.laex.in


 
     
RRP 2022 - Environment UPSC
Previous Year Questions
               
Answer: A Answer: B

Explanation: The Ministry of Agriculture has Explanation: Conservation Agriculture (CA)


issued a notification dated December 15, 2016 aims to achieve sustainable and profitable
inviting objections and suggestions from the agriculture and improve farmers’ livelihoods. CA
stakeholders on completely banning of 12 observes three main principles
pesticides/insecticides and phasing out of 6
• Minimal soil disturbance – (i.e., no
pesticides/insecticides.
tillage) through direct seed and/or
These pesticides include: Benomyl, Carbaryl, fertilizer placement.
Diazinon, Fenarimol, Fenthion, Linuron, MEMC,
• A permanent soil cover - (at least 30
Methyl Parthion, Sodium Cyanide, Thiometon,
percent) with crop residues and/or cover
Tridemorph, Trifluralin, Alachlor, Dichlorvos,
crops.
Phorate, Phosphamidon, Triazophos and
Trichlorfon. Hence, statement 1 is correct. • rotation of crops / Species diversification
- through varied crop sequences and
Educational objective: Carbofuran, methyl
associations involving at least three
parathion, prorate and triazophos are Pesticides
different crops
used in agriculture.

Form: Single answered based

Style: Description based question

Substance: Terminology based questions

24. With reference to the circumstances in


Indian agriculture, the concept of Educational objective: Adopting minimum
"Conservation Agriculture” assumes tillage, Avoiding the cultivation of plantation
significance. Which of the following fall crops, Using crop residues to cover soil surface
under the Conservation Agriculture? [2018] and Adopting spatial and temporal crop
sequencing/crop rotations fall under the
1. Avoiding the monoculture practices
Conservation Agriculture.
2. Adopting minimum tillage
3. Avoiding the cultivation of plantation crops Form: List based question
4. Using crop residues to cover soil surface
Style: Reference Based
5. Adopting spatial and temporal crop
sequencing/crop rotations Substance: Current affair-based questions
Select the correct answer using the code
25. Consider the following agricultural
given below:
practices: [2012]
a. 1,3 and 4 1. Contour bunding
b. 2,3,4 and 5 2. Relay cropping
c. 2,4 and 5 3. Zero tillage
d. 1,2,3 and 5
www.laexias.com Page 31 https://elearn.laex.in
 
     
RRP 2022 - Environment UPSC
Previous Year Questions
               
In the context of global climate change, Educational objective: Contour bunding, Relay
which of the above helps/help in carbon cropping AND Zero tillage helps in carbon
sequestration/storage in the soil? sequestration.

a. 1 and 2 only Form: List based question


b. 3 only
Style: Consider based question
c. 1, 2 and 3
d. None of them Substance: Cause and effect-based questions
Answer: C

Explanation: Carbon sequestration is the


3. Contemporary
process of capture and long-term storage of Environment Issues
atmospheric carbon dioxide to mitigate global
warming and to avoid dangerous impacts of 3.1. Environmental Pollution
climate change. 3.1.1. Air Pollution & Effects

soil is one of the largest reservoirs, where carbon 3.1.1.1. Industrial Pollution
could be restored. So, methods of soil
1. Magnetite particles, suspected to cause
conservation help in carbon sequestration.
neurodegenerative problems are
Contour bunding involves the construction of generated as environmental pollutants
banks along the contours. Terracing and from which of the following? (2021)
contour bunding which divide the hill slope into 1. Brakes of motor vehicles
numerous small slopes, check the flow of water, 2. Engines of motor vehicles
promote absorption of water by soil and save soil 3. Microwave stoves within homes
from erosion. Hence, statement 1 is correct 4. Power plants
5. Telephone lines
Relay cropping: Relay cropping involves
growing of two or more crops on the same field Select the correct answer using the code given
with the planting of the second crop after the below
first one has reached its reproductive stage. It
(a) 1, 2, 3 and 5only
saves soil from erosion. Hence, statement 2 is
(b) 1, 2 and 4 only
correct
(c) 3, 4 and 5 only
Zero Tillage farming is the method of farming (d) 1, 2, 3, 4 and 5
in which there is no tilling involved. Here, the Answer: D
soil is left undisturbed except to place the seeds
in the desired position to germinate. Hence, Explanation:
statement 3 is correct.
Magnetite particles can act as pollutants and led
Educational objective: To know about methods to neurodegenerative problems. Magnetic
that help in carbon sequestration in soil. particles can penetrate and damage the blood-
brain-barrier and led to the early development of
Alzheimer’s disease.
www.laexias.com Page 32 https://elearn.laex.in
 
     
RRP 2022 - Environment UPSC
Previous Year Questions
               
Whenever any magnetic component is used in 2. The copper slag can cause the leaching of
appliances, they generate magnetic flux which some heavy metals into the environment
produces magnetic particles. 3. They may release sulphur dioxide as a
pollutant
Option 1 is correct. The automotive industry
uses ceramic or ferrite magnets to make vehicles Select the correct answer using the code
safer. One of the most impressive uses is in the given below
Anti-lock Braking System (ABS) of vehicles. (a) 1 and 2 only
These use of magnets in motor vehicles can (b) 2 and 3 only
generate magnetic particles. (c) 1 and 3 only
Option 2 is correct. Engines of motor vehicles (d) 1, 2 and 3
can generate magnetic particles as they use high Answer: B
powered magnets which generate magnetic flux.
Explanation:
Option 3 is correct. In Microwave, a magnetic
Statement 1 is incorrect. In copper smelting
stirrer is used which can generate magnetic
process, first, the carbon (C) combusts with
particles.
oxygen (O2) in the air to produce carbon
Option 4 is correct. Power Plants releases metal monoxide (CO). Second, the carbon monoxide
content along with ash. Use of coal in power reacts with the ore and removes one of its
plants is a major source of combustion- oxygen atoms, releasing carbon dioxide. Hence,
associated magnetite fine particles. it does not release lethal quantity of CO in the
environment.
Option 5 is correct. Telephone line produces low
frequency, low energy electromagnetic field Statement 2 is correct. Copper smelters emit
which are the potential source of magnetite staggering amounts of toxic pollution i.e., lead,
particles. arsenic and selenium. These heavy metals and
particulate matter from copper smelters
Educational objective: Magnetite particles,
contaminate the environment downwind and
suspected to cause neurodegenerative problems
downstream.
can be generated from Brakes of motor vehicles,
Engines of motor vehicles, Microwave stoves Statement 3 is correct. Copper smelters are the
within homes, Power plants and Telephone lines. largest source of sulphur oxides (SOx) and trace
elements. Suspended particles have a pollution
Form: List based question
scope of 2-3 km, while Sulphur dioxide has a
Style: Which based questions range of as much as 15 km.

Substance: Cause and effect-based questions Educational objective: The copper slag can
cause the leaching of some heavy metals into the
2. Why is there a concern about copper
environment copper smelting plants they may
smelting plants? (2021)
release sulphur dioxide as a pollutant.
1. They may release lethal quantities of carbon
monoxide into the environment Form: Three statement Based question

www.laexias.com Page 33 https://elearn.laex.in


 
     
RRP 2022 - Environment UPSC
Previous Year Questions
               
Style: Why based questions generate power and Its use causes sulphur
emissions into the environment.
Substance: Species based question
Form: Three statement Based question
3. With reference to furnace oil, consider
the following statements: (2021) Style: Reference Based
1. It is a product of oil refineries Substance: Terminology based questions
2. Some industries use it to generate power
3. Its use causes sulphur emissions into the 4. “R2 Code of Practices” constitute a tool
environment available for promoting the adoption of
(2021)
Which of the statements given above are
correct? (a) Environmentally responsible practices in
the electronics recycling industry
(a) 1 and 2 only (b) Ecological management of ‘’Wetlands of
(b) 2 and 3 only International Importance” under the
(c) 1 and 3 only Ramsar Convention
(d) 1, 2 and 3 (c) Sustainable practices in the cultivation of
Answer: D agricultural crops in degraded lands
(d) ‘’Environmental Impact Assessment’’ in the
Explanation:
exploitation of natural resources
Statement 1 is correct. Furnace oil is a dark Answer: A
viscous residual fuel obtained by blending
heavier component of crude oil. It is a is a Explanation:
product of oil refineries. The R2 Standard was originally developed to
Statement 2 is correct. Furnace oil uses include prevent the misuse and improper recycling of
heating up homes and fuel trucks to lighting up electronic waste. The Sustainable Electronics
furnaces, producing steam for industrial uses Recycling International (SERI) institute defines
and for generating electric energy. It is produced the R2 Standard as, “the premier global
from the burnable liquids derived from crude oil environmental, worker health and safety
and is also called kerosene, home heating oil, standard for the electronics refurbishing and
diesel fuel or coal oil. recycling industry”. A goal of this standard is to
help prevent unnecessary recycling, and so
Statement 3 is correct. Sulphur is a natural a reuse business was born. Reuse refers to a
component in crude oil that is present in product that’s been used but still has life in it.
gasoline and diesel unless removed. Furnace oil
has high component of sulphur leading to its Educational objective: R2 Code of Practices
emission when burned. tool Environmentally responsible practices in the
electronics recycling industry.
Educational objective: Furnace oil is a product
of oil refineries, Some industries use it to Form: Single statement

Style: Consider based question

www.laexias.com Page 34 https://elearn.laex.in


 
     
RRP 2022 - Environment UPSC
Previous Year Questions
               
Substance: Species based question from 63 countries assessed) and the only city in
India to get this recognition so far.
5. Which one of the following is correct in
respect of the above statements? (2021) Educational objective: Arbor Day Foundation
Statement 1: The United Nations Capital and the FAO recognised Hyderabad as 2020 Tree
Development Fund (UNCDF) and the Arbor Day City of the World for its commitment to grow and
Foundation have recently recognized Hyderabad maintain the Urban forests.
as 2020 Tree City of the World Form: Two statements based
Statement 2: Hyderabad was selected for Style: Description based question
recognition for a year following its commitment
to grow and maintain the Urban forests. Substance: Current affair-based questions

(a) Both Statement 1 and Statement 2 are 6. Which of the following are the reasons/
correct and Statement 2 is the correct factors for exposure to benzene
explanation for Statement 1 pollution? [2020]
(b) Both Statement 1 and Statement 2 are 1. Automobile exhaust
correct but Statement 2 is not the correct 2. Tobacco smoke
explanation for Statement 1. 3. Wood burning
(c) Statement 1 is correct but Statement 2 is 4. Using varnished wooden furniture
not correct. 5. Using products made of polyurethane
(d) Statement 1 is not correct but Statement 2
Select the correct answer using the code
is correct.
given below:
Answer: D
a) 1, 2 and 3 only
Explanation: b) 2 and 4 only
c) 1, 3 and 4 only
Statement 1 is incorrect.
d) 1, 2, 3, 4 and 5
Hyderabad have been recognised as a 2020 Tree Answer: D
City of the World by the Arbor Day Foundation
and the Food and Agriculture Organization Explanation:
(FAO), [not United Nations Capital Development
Natural sources of benzene include volcanoes
Fund (UNCDF)].
and forest fires. Benzene is also a natural part of
Statement 2 is correct. crude oil, gasoline, and cigarette smoke. Some
industries use benzene to make other chemicals
It was given for its commitment to growing and that are used to make plastics, resins, and nylon
maintaining urban forests. and synthetic fibers. Benzene is also used to
Hyderabad earned recognition in the make some types of lubricants, rubbers, dyes,
Foundation’s second year of the programme detergents, drugs, and pesticides.
along with 51 other cities globally (120 cities Educational Objective: To know about the
causes of Benzene Pollution.

www.laexias.com Page 35 https://elearn.laex.in


 
     
RRP 2022 - Environment UPSC
Previous Year Questions
               
Style: Current affair based question Style: Consider based

Substance: Terminology based Substance:

Form: List based Form: Three statement based

7. Consider the following statements: [2020] 8. With reference to 'fly ash' produced by
1. Coal ash contains arsenic, lead and the power plants using coal as fuel,
mercury. which of the following statements is/are
2. Coal-fired power plants release sulphur correct? [2015]
dioxide and oxides of nitrogen into the 1) Fly ash can be used in the production of
environment. bricks for building construction.
3. High ash content is observed in Indian coal. 2) Fly ash can be used as a replacement for
Which of the statements given above is/are some of the Portland cement contents of
correct? concrete.
3) Fly ash is made up of silicon dioxide and
a) 1 only calcium oxide only, and does not contain
b) 2 and 3 only any toxic elements.
c) 3 only
Select the correct answer using the code
d) 1, 2 and 3
given below:
Answer: D
a. 1 and 2 only
Explanation: b. 2 only
c. 1 and 3 only
Coal ash, also referred to as coal combustion
d. 3 only
residuals or CCRs, is produced primarily from
the burning of coal in coal-fired power plants. Answer: A
Coal ash contains contaminants like mercury,
Explanation:
cadmium and arsenic.
Fly Ash: It is a by-product from burning of coal
Excessive burning of coal, wood, kerosene,
in electric power generating plants. It is called fly
petrol etc. release toxic gases such as SO2
ash because it is transported from the
(sulphur dioxide), NOx (oxide of nitrogen), CO
combustion chamber by exhaust gases. It is
(carbon monoxide) and hydrocarbons in the air.
collected from the exhaust gases by electrostatic
These gases are also emitted by industries,
precipitators or bag filters.
power plants, automobiles and aircrafts.
Composition: Fly ash includes substantial
India's domestic coal reserves have a high ash
amounts of silicon dioxide (SiO2), aluminium
content-up to 40 to 45 percent. As a result, more
oxide (Al2O3), ferric oxide (Fe2O3) and calcium
coal is required to generate one kilowatt hour of
oxide (CaO).
electricity. This in turn means higher emissions.

Educational Objective: To know about the


thermal pollution.

www.laexias.com Page 36 https://elearn.laex.in


 
     
RRP 2022 - Environment UPSC
Previous Year Questions
               
Fly Ash Utilisation: NTPC has collaborated with Educational Objective: To know about
Cement manufacturers around the country to pollution caused by industrial sector.
supply Fly Ash.
Style: Which based
To promote the use of Fly Ash bricks in building
Substance: Cause and effect based
construction, NTPC has set up Fly Ash brick
manufacturing Plants at its Coal based Thermal Form: List based
Power Plants.
3.1.1.2. Vehicular Pollution
Fly ash can be used as a replacement for some
10. Acid rain is caused by the pollution of
of the Portland cement contents of concrete.
environment by [2013]
Educational Objective: To know about fly ash a. Carbon dioxide and nitrogen
and its impact on environment. b. Carbon monoxide and carbon dioxide
Style: Reference based question c. Ozone and carbon dioxide
d. Nitrous oxide and sulphur dioxide
Substance: Terminology based question
Answer: D
Form: Three statement
Explanation:
9. Which of the following are some
Burning of fossil fuel like coal, diesel etc…
important pollutants released by steel
releases pollutants like sulphur dioxide (SO2)
industry in India? [2014]
and nitrogen dioxide (NO2) into atmosphere.
1) Oxides of sulphur
2) Oxides of nitrogen When these pollutants react with water vapour
3) Carbon monoxide in atmosphere they convert into sulphuric acid
4) Carbon dioxide and nitric acid.

Select the correct answer using the code These acids come down along with rain as Acid
given below. rain damaging crops and leads to corrosion of
buildings and monuments.
a. 1, 3 and 4 only
b. 2 and 3 only Educational Objective: To know about Acid
c. 1 and 4 only rain impact on environment
d. 1, 2, 3 and 4
Style: Current affairs based
Answer: D
Substance: Cause and Effect based
Explanation:
Form: Single answered
In steel furnace, coke reacts with iron ore,
releasing iron and generating Carbon Monoxide 11. Photochemical smog is a resultant of the
(CO) and Carbon dioxide (CO2) gases. Due to use reaction among [2013]
of coal, pollutants such as SOx and NOx are a. NO2, O3, and peroxyacetyl nitrate in the
released. presence of sunlight

www.laexias.com Page 37 https://elearn.laex.in


 
     
RRP 2022 - Environment UPSC
Previous Year Questions
               
b. CO, O2, and peroxyacetyle nitrate in the
presence of sunlight
c. CO, CO2, and NO2, at low temperature
d. High concentration of NO2, O3, and CO in
the evening
Answer: A Educational Objective: To know about basic
Explanation: concepts related to photo chemical smog.

Formation of photochemical smog. Style: Current affair based

When fossil fuels are burnt, a variety of Substance: Cause and effect based
pollutants are emitted into the earth’s Form: Single answered
troposphere. Two of the pollutants that are
emitted are hydrocarbons (unburnt fuels) and 12. Brominated flame retardants are used in
nitric oxide (NO). When these pollutants build many household products like mattresses
up to sufficiently high levels, a chain reaction and upholstery, why is there some
occurs from their interaction with sunlight in concern about their use? [2014]
which NO is converted into nitrogen dioxide 1) They are highly resistant to degradation in
(NO2). This NO2 in turn absorbs energy from the environment.
sunlight and breaks up into nitric oxide and free 2) They are able to accumulate in humans and
oxygen atom. animals.

NO2(g) NO(g) + ÆO(g) ------i Select the correct answer using the code
given below.
Oxygen atoms are very reactive and combine
with the O2 in air to produce ozone. a. 1 only
b. 2 only
O(g) + O2 (g) Æ O3 (g) -------ii
c. Both 1 and 2
The ozone formed in the above reaction (ii) reacts d. Neither 1 nor 2
rapidly with the NO(g) formed in the reaction (i) Answer: C
to regenerate NO2. NO2 is a brown gas and at
sufficiently high levels can contribute to haze. Explanation:

NO (g) + O3 (g) → NO2 (g) + O2 (g) Brominated flame retardants are mixtures of
man-made chemicals that are added to various
Ozone is a toxic gas and both NO2 and O3 are products to make them less flammable. They are
strong oxidizing agents and can react with the used commonly in plastics, textiles and
unburnt hydrocarbons in the polluted air to electrical/electronic equipment. There are five
produce chemicals such as formaldehyde, main classes of BFRs, listed here with their
acrolein and peroxyacetyl nitrate (PAN). common uses:

www.laexias.com Page 38 https://elearn.laex.in


 
     
RRP 2022 - Environment UPSC
Previous Year Questions
               
• Polybrominated diphenyl ethers (PBDEs) – active muscles). When carbon monoxide binds to
plastics, textiles, electronic castings, haemoglobin, it cannot be released as easily as
circuitry. oxygen. The slow-release rate of carbon
• Hexabromocyclododecanes (HBCDDs) – monoxide causes an accumulation of CO-bound
thermal insulation in the building industry. haemoglobin molecules as exposure to carbon
• Tetrabromobisphenol A (TBBPA) and other monoxide continues. Because of this, fewer
phenols – printed circuit boards, haemoglobin particles are available to bind and
thermoplastics (mainly in TVs). deliver oxygen, thus causing the gradual
• Polybrominated biphenyls (PBBs) – suffocation associated with carbon monoxide
consumer appliances, textiles, plastic foams. poisoning.
• Other brominated flame retardants.
Educational Objective: To know about Various
Properties: Resistant to degradation, bio- impact of Carbon monoxide.
accumulative and toxic to both humans and the
environment. Style: What based

Educational Objective: To know about Substance: Cause and effect


Brominated Flame Retardants Form: Single answered
Style: Why based Carbon Pollution
Substance: Cause and effect based 14. Which one of the following statements
Form: Two statement question best describes the term ‘Social Cost of
Carbon’? [2020]
13. Excessive release of the pollutant carbon
It is a measure, in monetary value, of the
monoxide (CO) into the air may produce
a condition in which oxygen supply in a) long-term damage done by a tonne of CO2
the human body decreases. What causes emissions in a given year.
this condition? [2010] b) requirement of fossil fuels for a country to
a. When inhaled into the human body, CO is provide goods and services to its citizens,
converted into CO2 based on the burning of those fuels.
b. The inhaled CO has much higher affinity for c) efforts put in by a climate refugee to adapt
haemoglobin as compared to oxygen to live in a new place
c. The inhaled CO destroys the chemical d) contribution of an individual person to the
structure of haemoglobin. carbon footprint on the planet Earth.
d. The inhaled CO adversely affects the Answer: A
respiratory centre in the brain
Explanation:
Answer: B
OECD defines Social Cost of Carbon as the net
Explanation: Normally, oxygen would bind to present value of climate change impacts over the
haemoglobin in the lungs and be released in next 100 years (or longer) of one additional
areas with low oxygen partial pressure (e.g., tonne of carbon emitted to the atmosphere

www.laexias.com Page 39 https://elearn.laex.in


 
     
RRP 2022 - Environment UPSC
Previous Year Questions
               
today. It is a long-term damage done due to without heating the aggregates using cationic
emissions. bitumen emulsion.

Educational Objective: To know about the The usage of Cold mix bitumen Emulsion in
effects of the carbon pollution India is a sustainable technology and ideal for
India as use of bitumen emulsion negates the
Style: Which based
heating of aggregate and binder.
Substance: terminology based
Copper slag is a by-product obtained during
Form: single answered smelting and refining of copper. Copper slag is a
non-hazardous, non-toxic material. This eco-
15. In rural road construction, the use of
friendly industrial by-product is being used in
which of the following is preferred for
government road projects.
ensuring environmental sustainability or
to reduce carbon footprint? [2020] Educational Objective: To know about the
environmentally sustainable materials used in
1. Copper slag
the road construction
2. Cold mix asphalt technology
3. Geotextiles Style: Current Affairs
4. Hot mix asphalt technology
5. Portland cement Substance: Environmental policies and
initiatives
Select the correct answer using the code
given below: Form: List based

a) 1, 2 and 3 only Agricultural Pollution


b) 2, 3 and 4 only
16. Consider the following; [2019]
c) 4 and 5 only
d) 1 and 5 only 1) Carbon monoxide
2) Methane
Answer: A
3) Ozone
Explanation: 4) Sulphur dioxide.

Coir is a natural fibre. Government has allowed Which of the above are released into
the use of coir-based geotextiles for construction atmosphere due to the burning of crop/
of rural roads under the Pradhan Mantri Gram biomass residue?
Sadak Yojana in 2020. a. 1 and 2 only
In India for decades Hot Mix Asphalt with b. 2, 3 and 4 only
bitumen aggregate has been used as Pavement c. 1 and 4 only
mixes. The wide use of hot mix technology leads d. 1, 2, 3 and 4
to environmental pollution as these plants emit Answer: D
a huge amount of Greenhouse gases. Cold Mix
Explanation: Open biomass burning, which
on the other hand is a pavement technology
refers to burning of forests, savanna/grasslands

www.laexias.com Page 40 https://elearn.laex.in


 
     
RRP 2022 - Environment UPSC
Previous Year Questions
               
and crop residue, releases large amounts of and end up in the seas. They do not degrade
gases such as Nitrogen Oxides (NOx), CO, over time and can transport toxic chemicals into
methane (CH4), non-methane hydrocarbon marine organisms. Microbeads can have a
(NMHCs), carbon dioxide (CO2) and particulate damaging effect on marine life, the environment
matter. and human health. This is due to their
composition, ability to adsorb toxins and
Ozone is released as secondary pollutant in the
potential to transfer it up the marine food chain.
burning of crop/biomass
Educational Objective: To know about
Educational Objective: To know about biomass
Microbeads impact on environment
burning and its impact on environment.
Style: Why based
Style: Consider based
Substance: Terminology based
Substance: Cause and effect based
Form: single answered
Form: list based
i. Water Pollution & Effects 18. Biological Oxygen Demand (BOD) is a
standard criterion for [2017]
17. Why is there a great concern about the
a. Measuring oxygen levels in blood
‘microbeads’ that are released into
b. Computing oxygen levels in forest
environment? [2019]
ecosystems
a. They are considered harmful to marine c. Pollution assay in aquatic ecosystems
ecosystems. d. Assessing oxygen levels in high altitude
b. They are considered to cause skin cancer in regions.
children.
Answer: C
c. They are small enough to be absorbed by
crop plants in irrigated fields. Explanation:
d. They are often found to be used as food
Biochemical oxygen demand (BOD, also called
adulterants.
biological oxygen demand) is the amount of
Answer: A dissolved oxygen (DO) needed (i.e., demanded)
by aerobic biological organisms to break down
Explanation: Microbeads are manufactured
organic material present in a given water sample
solid plastic particles of less than one millimeter
at certain temperature over a specific time
in their largest dimension. They are most
period.
frequently made of polyethylene but can be of
other petrochemical plastics such as The BOD value is most commonly expressed in
polypropylene and polystyrene. These tiny pieces milligrams of oxygen consumed per litre of
of plastic are added to rinse-off products such as sample during 5 days of incubation at 20 °C and
face scrubs, toothpastes and shower gels. The is often used as a surrogate of the degree of
small spherical beads, designed to help with organic pollution of water.
exfoliation and then be washed down the drain,
often slip through waste-water treatment plants

www.laexias.com Page 41 https://elearn.laex.in


 
     
RRP 2022 - Environment UPSC
Previous Year Questions
               
BOD can be used as a gauge of the effectiveness • Physical contaminants primarily impact the
of wastewater treatment plants. physical appearance or other physical
properties of water. Examples of physical
Educational Objective: To know about
contaminants are sediment or organic
Biological Oxygen Demand
material suspended in the water of lakes,
Style: Descriptive based rivers and streams from soil erosion.
• Chemical contaminants: These
Substance: Terminology based
contaminants may be naturally occurring or
Form: Single answered man-made. Examples of chemical
contaminants include nitrogen, bleach,
19. Which of the following can be found as arsenic, salts (fluoride), pesticides, metals,
pollutants in the drinking water in some toxins produced by bacteria, and human or
parts of India? [2013] animal drugs.
1) Arsenic • Biological contaminants are organisms in
2) Sorbitol water. They are also referred to as microbes
3) Fluoride or microbiological contaminants. Examples
4) Formaldehyde of biological or microbial contaminants
5) Uranium include bacteria, viruses, protozoan, and
Select the correct answer using the codes parasites.
given below. • Radiological contaminants are chemical
elements with an unbalanced number of
a. 1 and 3 only protons and neutrons resulting in unstable
b. 2, 4 and 5 only atoms that can emit ionizing radiation.
c. 1, 3 and 5 only Examples of radiological contaminants
d. 1, 2, 3, 4 and 5 include cesium, plutonium and uranium.
Answer: C Sorbitol is sugar alcohol (used as artificial
sweetener) and Formaldehyde (used in coating,
Explanation: Contaminants are anything
etc) are not water pollutants.
physical, chemical, biological, or matter in
water. Drinking water may reasonably be Educational Objective: To know about various
expected to contain at least small amounts of pollutants contamination in drinking water
some contaminants. Some drinking water
contaminants may be harmful if consumed at Style: Which based
certain levels in drinking water while others may Substance: causes and effect based
be harmless. The presence of contaminants does
not necessarily indicate that the water poses a Form: List based
health risk.

The following are general categories of drinking


water contaminants and examples of each:

www.laexias.com Page 42 https://elearn.laex.in


 
     
RRP 2022 - Environment UPSC
Previous Year Questions
               
ii. Soil Pollution & Effects iii. Pollution Control Techniques

20. Consider the following statements: [2019] 21. In the context of solving pollution
1) Agricultural soils release nitrogen oxides problems, what is/are the advantage/
into environment. advantages of bioremediation technique?
2) Cattle release ammonia into environment. [2017]
3) Poultry industry releases reactive nitrogen 1) It is a technique for cleaning up pollution by
compounds into environment. enhancing the same biodegradation process
Which of the statement given above is/are that occurs in nature.
correct? 2) Any contaminant with heavy metals such as
cadmium and lead can be readily and
a. 1 and 3 only completely treated by bioremediation using
b. 2 and 3 only microorganisms.
c. 2 only 3) Genetic engineering can be used to create
d. 1, 2 and 3 microorganisms specifically designed for
Answer: D bioremediation.
Select the correct answer using the code
Explanation:
given below:
Agricultural soils contributed to over 70% of N2O
a. 1 only
emissions from India in 2010, followed by waste
b. 2 and 3 only
water (12%) and residential and commercial
c. 1 and 3 only
activities (6%). Since 2002, N2O has replaced
d. 1, 2 and 3
methane as the second largest Greenhouse Gas
(GHG) from Indian agriculture. Answer: C

Livestock on farms produce a lot of manure and Explanation: Bioremediation is a process used
thus a lot of ammonia, which is harmful for the to treat contaminated media, including water,
environment and reduces biodiversity. Cattle soil and subsurface material, by altering
account for 80% of the ammonia production. environmental conditions to stimulate growth of
microorganisms and degrade the target
The poultry industry recorded an excretion of
pollutants.
reactive nitrogen compounds of 0.415 tonnes in
2016. That is anticipated to increase to 1.089 Toxic metals like Cadmium and Lead cannot be
tonnes by 2030. treated with this technique.

Educational Objective: To know about It is possible to create microorganisms with the


agriculture sector contribution to pollution approach of Genetic Engineering for
bioremediation.
Style: Consider based
Educational Objective: To know about
Substance: Causes and effect based
Bioremediation process to control pollution
Form: Three statement question
Style: What based

www.laexias.com Page 43 https://elearn.laex.in


 
     
RRP 2022 - Environment UPSC
Previous Year Questions
               
Substance: Terminology based b. Global Environmental Issues
i. Greenhouse effect
Form: three statement question
23. In the context of which of the following
22. In the cities of our country, which among
do some scientists suggest the use of
the following atmospheric gases are
cirrus cloud thinning technique and the
normally considered in calculating the
injection of Sulphate aerosol into
value of Air Quality Index? [2016]
stratosphere? [2019]
1) Carbon dioxide
a. Creating the artificial rains in some regions
2) Carbon monoxide
b. Reducing the frequency and intensity of
3) Nitrogen dioxide
tropical cyclones
4) Sulphur dioxide
c. Reducing the adverse effects of solar wind
5) Methane
on the earth
Select the correct answer using the code d. Reducing the global warming
given below.
Answer: D
a. 1, 2 and 3 only
Explanation:
b. 2, 3 and 4 only
c. 1, 4 and 5 only Cirrus Clouds:
d. 1,2, 3, 4 and 5
Cirrus clouds reflect sunlight and absorb
Answer: B warming infrared radiation resulting in a net
warming effect on the climate. Therefore,
Explanation:
thinning or removing these clouds will reduce
Air Quality Index gives real time data about level their heat trapping capacity and cools down the
of pollutants in the air and inform people about Earth system by trapping only a fraction of heat.
possible impacts on health.
Solar Winds have hardly any significant impact
It classifies air quality simply as good, on earth hence it is incorrect.
satisfactory, moderately polluted, poor, very
poor, and severe. Reducing the frequency and intensity of
tropical cyclones requires altering ocean water
Pollutants measured by AQI: Nitrogen oxide, temperatures which is humanly impossible and
Sulphur dioxide, PM2.5, PM 10, Ozone, Carbon hence is incorrect.
monoxide, Ammonia, Lead.
Artificial rains are created by spraying Dry Ice
Educational Objective: To know about Air (solid carbon dioxide), Silver Iodide, Salt powder
Quality Index. etc. on the rain bearing clouds. Hence it is
incorrect.
Style: Current affairs based
Educational Objective: Test candidate's
Substance: Environmental policies and
knowledge of various methods being suggested
initiatives
to reduce the effect of global warming.
Form: List based

www.laexias.com Page 44 https://elearn.laex.in


 
     
RRP 2022 - Environment UPSC
Previous Year Questions
               
Style: Current affairs based 25. The scientific view is that the increase in
global temperature should not exceed 2°C
Substance: Terminology based
above pre-industrial level. If the global
Form: Single answered temperature increases beyond 3°C above
the pre-industrial level, what can be its
24. In the context of mitigating the
possible impact/impacts on the world?
impending global warming due to
[2014]
anthropogenic emissions of carbon
dioxide, which of the following can be the 1) Terrestrial biosphere tends toward a net
potential sites for carbon sequestration? carbon source.
[2017] 2) Widespread coral mortality will occur.
3) All the global wetlands will permanently
1) Abandoned and uneconomic coal seams
disappear.
2) Depleted oil and gas reservoirs
4) Cultivation of cereals will not be possible
3) Subterranean deep saline formations
anywhere in the world.
Select the correct answer using the code
Select the correct answer using the code
given below
given below
a. 1 and 2 only
a. l only
b. 3 only
b. 1 and 2 only
c. 1 and 3 only
c. 2, 3 and 4 only
d. 1,2 and 3
d. 1,2,3 and 4
Answer: D
Answer: B
Explanation: Carbon sequestration or carbon
Explanation: Impact of rise in temperature
dioxide removal from the atmosphere will slow
above 2 degrees from pre-industrial levels:
down the atmospheric CO2 pollution and
Increased air temperature; shifts in
mitigate global warming. The carbon that is
precipitation; increased frequency of storms,
captured can be stored in geological formations
droughts, and floods; increased atmospheric
such as deep saline aquifers, basalt formations,
carbon dioxide concentration; and sea level rise
uneconomic coal seams and depleted oil or gas
substantially threatens the life of wetlands but
reservoirs.
not permanently.
Educational Objective: To learn various
methods that can be employed to reduce the Due to global warming, the waters are getting
release of various Greenhouse Gases. more acidic (becoming carbon source) and
posing threats to the coral ecosystem. Moreover,
Style: Which based
the glaciers across the world are melting due to
Substance: Terminology based which sea levels are rising. Rising sea levels lead
to the coral reefs drowning and not getting
Form: List based
enough sunlight which leads to their slow
growth.

www.laexias.com Page 45 https://elearn.laex.in


 
     
RRP 2022 - Environment UPSC
Previous Year Questions
               
Statement 4 is also extreme which says Form: Singe answered
cultivation of cereals will not be possible and ii. Depletion of Ozone Layer
can be eliminated.
27. Which one of the following is associated
Educational Objective: Understand the impact
with the issue of control and phasing out
of Global warming and climate change.
of the use of ozone-depleting
Style: Current affairs based substances? [2015]

Substance: causes and effects a. Bretton Woods Conference


b. Montreal Protocol
Form: Four statement based c. Kyoto Protocol
d. Nagoya Protocol
26. The increasing amount of carbon dioxide
in the air is slowly raising the Answer: B
temperature of the atmosphere, because
Explanation:
it absorbs: [2012]
a. The water vapour of the air and retains its Bretton Woods Conference: This conference led
heat to the establishment of the World Bank and IMF.
b. The ultraviolet part of the solar radiation Montreal Protocol: The Montreal Protocol is an
c. All the solar radiations international treaty designed to protect the
d. The infrared part of the solar radiation ozone layer by phasing out the production of
Answer: D numerous substances that are responsible for
ozone depletion.
Explanation: Molecules of carbon dioxide (CO2)
can absorb energy from infrared (IR) radiation. Kyoto Protocol: The Kyoto Protocol is an
This ability to absorb and re-emit infrared international treaty which extends the 1992
energy is what makes CO2 an effective heat- United Nations Framework Convention on
trapping greenhouse gas. Not all gas molecules Climate Change (UNFCCC) that commits state
are able to absorb IR radiation. For example, parties to reduce greenhouse gas emissions.
Nitrogen (N2) and Oxygen (O2), which make up
Nagoya Protocol: The Nagoya Protocol is about
more than 90% of Earth's atmosphere, do not
Access to Genetic Resources and the Fair and
absorb infrared photons. CO2 molecules can
Equitable Sharing of Benefits Arising from their
vibrate in ways that simpler nitrogen and oxygen
Utilization to the Convention on Biological
molecules cannot, which allows CO2 molecules
Diversity.
to capture the IR photons.
Educational Objective: To learn and
Educational Objective: Test candidate's
differentiate various international treaties signed
knowledge of harmful effects of carbon dioxide
in the context of saving mother earth.
on the environment.
Style: Which based
Style: Descriptive based
Substance: International conventions
Substance: Causes and effect based

www.laexias.com Page 46 https://elearn.laex.in


 
     
RRP 2022 - Environment UPSC
Previous Year Questions
               
Form: Single answered 29. The formation of ozone hole in the
Antarctic region has been a cause, of
28. Consider the following statements: [2012]
concern. What could be the reason for
Chlorofluorocarbons, known as ozone- the formation of this hole? [2011]
depleting substances, are used.
a. Presence of prominent tropospheric
1) In the production of plastic foams turbulence; and inflow of
2) In the production of tubeless tyres chlorofluorocarbons
3) In cleaning certain electronic components b. Presence of prominent polar front and
4) As pressurizing agents in aerosol cans stratospheric clouds; and inflow of
Which of the statements given above is/are chlorofluorocarbons
correct? c. Absence of polar front and stratospheric
clouds; and inflow of methane and
a. 1, 2 and 3 only chlorofluorocarbons.
b. 4 only d. Increased temperature at polar region due
c. 1, 3 and 4 only to global warming
d. 1, 2, 3 and 4
Answer: B
Answer: C
Explanation:
Explanation: CFCs are widely used in four
areas: aerosol propellants, solvents in the Polar Stratospheric Clouds or nacreous clouds
cleaning of electronic components and metals, contain water, nitric acid and/or sulfuric acid.
refrigerants, and the blowing agents in foam They are formed mainly during the event of polar
plastic manufacturing. In aerosols, CFCs such vertex in winter; more intense at south pole. The
as trichlorofluoro methane (CCl3F, CFC-11) and Cl-catalyzed ozone depletion is dramatically
dichlorodifluoromethane (CCl2F2) were normally enhanced in the presence of polar stratospheric
packed with materials like paint, insecticide, or clouds (PSCs). Polar Stratospheric Clouds
cosmetic preparations in pressurized canisters. convert “reservoir” compounds into reactive free
radicals (Cl and ClO). These free radicals deplete
Tubeless tyre has no component which is a ozone
Greenhouse Gas.
Educational Objective: Test candidate's
Educational Objective: To learn different understanding of Ozone hole formation.
sources of Greenhouse Gases.
Style: What based
Style: Consider based questions
Substance: Cause and effect based question
Substance: Cause and effect
Form: Single answered
Form: List based

www.laexias.com Page 47 https://elearn.laex.in


 
     
RRP 2022 - Environment UPSC
Previous Year Questions
               
iii. Ocean acidification phytoplankton enters the atmosphere and reacts
to make sulphuric acid, which clumps into
30. The acidification of oceans is increasing.
aerosols, or microscopic airborne particles.
Why is this phenomenon a cause of
Aerosols seed the formation of clouds, which
concern? [2012]
help cool the Earth by reflecting sunlight. But,
1) The growth and survival of calcareous in acidified ocean water, phytoplankton
phytoplankton will be adversely affected. produces fewer DMS. This reduction of Sulphur
2) The growth and survival of coral reefs will may lead to decreased cloud formation, raising
be adversely affected. global temperatures.
3) The survival of some animals that have
phytoplanktonic larvae will be adversely Educational Objective: To know about Ocean
affected. acidification
4) The cloud seeding and formation of clouds Style: Why based/reference based
will be adversely affected.
Substance: Cause and effect based
Which of statements given above is / are
correct? Form: Four statement question

a. 1,2 and 3 only iv. Dumping of Hazardous wastes


b. 2 only
31. Due to improper / indiscriminate
c. 1 and 3 only
disposal of old and used computers or
d. 1,2,3 and 4
their parts, which of the following are
Answer: D released into the environment as
e-waste? [2013]
Explanation:
1) Beryllium
Ocean acidification means reduction in the pH of
2) Cadmium
the ocean over an extended period of time, due
3) Chromium
to absorption of carbon dioxide (CO2) from the
4) Heptachlor
atmosphere. The increase in atmospheric CO2
5) Mercury
levels leads to an increase in the concentration
6) Lead
of carbonic acid and bicarbonate ions, causing a
7) Plutonium
decrease in the concentration of carbonate ions.
The decrease in the amount of carbonate ions Select the correct answer using the codes
available makes it more difficult for marine given below.
calcifying organisms, such as coral (calcareous a. 1, 3, 4, 6 and 7 only
corals) and some plankton (calcareous b. 1, 2, 3, 5 and 6 only
plankton), to form biogenic calcium carbonate. c. 2, 4, 5 and 7 only
The majority of Sulphur in the atmosphere is d. 1, 2, 3, 4, 5, 6 and 7
emitted from the ocean, often in the form of
dimethyl sulfide (DMS) produced by
phytoplankton. Some of the DMS produced by

www.laexias.com Page 48 https://elearn.laex.in


 
     
RRP 2022 - Environment UPSC
Previous Year Questions
               
Answer: B
4. Environmental
Explanation: Plutonium is a radioactive
substance and it is highly unlikely that it is an
Management
ingredient in making electronic goods.
4.1. Environmental Legislations &
Heptachlor is an organochlorine compound that
Initiatives
was used as an insecticide. And hence does not
contribute to e-waste. 4.1.1. Waste Management related Acts
4.1.1.1. Solid Waste Acts/Policies
Lead is used in glass panels and gaskets in
computer monitors. 1. As per the Solid Waste Management
Rules, 2016 in India, which one of the
Cadmium occurs in SMD chip resistors, infrared
following statements is correct? [2019]
detectors, and semiconductor chips.
(a) Waste generator has to segregate waste into
Beryllium is commonly found on motherboards five categories.
and finger clips. (b) The Rules are applicable to notified urban
local bodies, notified towns and all
Chromium VI (Hexavalent Chromium) is used
industrial townships only.
as a corrosion protector of untreated and
(c) The rules provide for exact and elaborate
galvanized steel plates and as a decorative or
criteria for the identification of sites for
hardener for steel housings Plastics.
landfills and waste processing facilities
Mercury is used in thermostats, sensors, relays,
(d) It is mandatory on the part of waste
switches, medical equipment, lamps, mobile
generator that the waste generated in one
phones and batteries.
district cannot be moved to another district.
Educational Objective: To know about E
Answer: C
wastes related pollution
Explanation:
Style: Which based
Solid Waste Management Rules 2016
Substance: Cause and effect based
These rules replace the Municipal Solid Wastes
Form: List based
(Management and Handling) Rules, 2000, are
now applicable beyond municipal areas and
have included urban agglomerations, census
towns, notified industrial townships etc.

The new rules have mandated the source


segregation of waste in order to channelize the
waste to wealth by recovery, reuse and recycle.

Waste generators would now have to segregate


waste into three streams -Biodegradables, Dry
(Plastic, Paper, metal, Wood, etc.) and Domestic

www.laexias.com Page 49 https://elearn.laex.in


 
     
RRP 2022 - Environment UPSC
Previous Year Questions
               
Hazardous waste (diapers, napkins, mosquito consumer has used and disposed them. India
repellents, cleaning agents etc.) before handing first introduced EPR to manage electronic-waste
it over to the collector. in 2012.

Rules specifies criteria for site selection, India extended EPR to plastic manufacturers
development of facilities at the sanitary landfills, after the Plastic Waste Management Rules 2016
specifications for land filling operations and (PWMR) which were notified in 2016.
closure on completion of land filling, pollution
Educational Objective: to know about the
prevention, Closure and Rehabilitation of Old
regulation of e-Waste Management in India
Dumps, specifies Criteria for special provisions
for hilly areas. So, C is correct. Style: Which based

There is no mention of district waste Substance: Environmental laws/ Terminology


management requirement. based

Educational Objective: to know about the Form: Single answered


regulation of Solid Waste Management in India
4.1.2. Pollution related Acts/Policies
Style: Which based 4.1.2.1. Environment Acts/Policies

Substance: Environmental laws 3. Consider the following statements: [2020]

Form: Single answered 1. 36% of India’s districts are classified as


“overexploited” or “critical” by the Central
4.1.1.2. E-Waste Acts/Policies
Ground Water Authority (CGWA).
2. In India, 'extended producer 2. CGWA was formed under Environment
responsibility' was introduced as an (Protection) Act.
important feature in which of the 3. India has the largest area under
following? [2019] groundwater irrigation in the world.

a. The Bio-medical Waste (Management and Which of the statements given above is/are
Handling) Rules, 1998 correct?
b. The Recycled Plastic (Manufacturing and
a) 1 only
Usage) Rules, 1999
b) 2 and 3 only
c. The e-Waste (Management and Handling)
c) 2 only
Rules, 2011
d) 1 and 3 only
d. The Food Safety and Standard Regulations,
2011 Answer: B

Answer: C Explanation:

Explanation: The Central Ground Water Authority (CGWA)


was formed under the Environment (Protection)
Extended Producer Responsibility makes the
Act. 1986.
Producer of e-waste responsible for collection of
E-waste, recycle and for its exchange after the

www.laexias.com Page 50 https://elearn.laex.in


 
     
RRP 2022 - Environment UPSC
Previous Year Questions
               
As per assessment carried out by CGWB in Answer: A
association with the State ground water
Explanation:
organisations in 5723 ground water assessment
units (i.e. blocks, mandals, talukas etc), the • National Policy on Biofuels 2018 identified
number of over-exploited, critical and semi following raw materials for biofuel
critical assessment units is given as generation:

Over-exploited - 839 • Molasses, Sugarcane juice, biomass in form


of grasses,
Critical - 226
• agriculture residues (Rice straw, cotton
Semi-critical – 550
stalk, corn cobs, saw dust, bagasse etc.).
The countries with the largest extent of areas
• sugar containing materials like sugar beet,
equipped for irrigation with groundwater, in
sweet sorghum, etc.
absolute terms, are India (39 million ha), China
(19 million ha) and the USA (17 million ha). • starch containing materials such as corn,
cassava, rotten potatoes etc.,
Educational Objective: To know about the
ground water distribution in India. • Damaged food grains like wheat, broken
rice etc.
Style: Consider based
Horse gram and Ground nut seeds are not there
Substance: Environmental laws, initiatives and
in the National Policy on Bio fuels.
policies
Educational Objective: To know the sources
Form: Three statement question
that are used in the production of bio fuels.
4. According to India’s National Policy on
Style: Which based
Biofuels, which of the following can be
used as raw materials for the production Substance: Environmental policies
of biofuels? [2020]
Form: List based
1. Cassava
2. Damaged wheat grains 5. Consider the following statements: [2019]
3. Groundnut seeds The Environment Protection Act, 1986 empowers
4. Horse gram the Government of India to
5. Rotten potatoes
6. Sugar beet 1) State the requirement of public
participation in the process of
Select the correct answer using the code
environmental protection, and the
given below:
procedure and manner in which it is
a) 1,2,5 and 6 only sought.
b) 1,3,4 and 6 only 2) Lay down the standards for emission or
c) 2,3,4 and 5 only discharge of environmental pollutants from
d) 1,2,3,4,5 and 6 various sources.

www.laexias.com Page 51 https://elearn.laex.in


 
     
RRP 2022 - Environment UPSC
Previous Year Questions
               
Which of the statements given above is/are Style: Consider based
correct?
Substance: Environmental laws
(a) 1 only
Form: Two statement question
(b) 2 only
(c) Both 1 and 2 6. The Genetic Engineering Appraisal
(d) Neither 1 nor 2 Committee is constituted under the
Answer: B [2015]
(a) Food Safety and Standards Act, 2006
Explanation: In the wake of the Bhopal tragedy,
(b) Geographical Indications of Goods
the government of India enacted the
(Registration and Protection) Act, 1999
Environment (Protection) Act of 1986.
(c) Environment (Protection) Act, 1986
It empowers the central government to take (d) Wildlife (Protection) Act, 1972
measures by setting standards for protecting Answer: C
and improving the quality of the environment
and preventing, controlling and abating Explanation: The Genetic Engineering Appraisal
environmental pollution. Committee (GEAC) was constituted under the
Ministry of Environment, Forest and Climate
The Central government is also empowered to: Change (MoEFCC) as the apex body under the
• Plan and Execute a nation-wide ‘Rules for Manufacture, Use, Import, Export and
programme for the prevention, control Storage of Hazardous Microorganisms /
and abatement of environmental Genetically Engineered Organisms or Cells 1989’
pollution. in accordance with the Environment
Protection Act, 1986.
• Lay down standards for the quality of
environment in its various aspects. Educational Objective: To know about the
Environmental protection laws of India
• Lay down standards for emission or
discharge of environmental pollutants Style: Current affairs based
from various sources. Substance: Environmental laws
• The restriction of areas in which any Form: single answered
industries, operations or processes or
class of industries, operations or 7. The National Green Tribunal Act, 2010
processes shall/ shall not be carried out was enacted in consonance with which of
subject to certain safeguards. the following provisions of the
Constitution of India? [2012]
The act does not mention about requirement of
1) Right to healthy environment, construed as
public participation.
a part of Right to life under Article 21.
Educational Objective: To know about the 2) Provision of grants for raising the level of
Environmental legislations of India administration in the Scheduled Areas for

www.laexias.com Page 52 https://elearn.laex.in


 
     
RRP 2022 - Environment UPSC
Previous Year Questions
               
the welfare of Scheduled Tribes under Select the correct answer using the code
Article 275(1). given below
Which of the statements given above is/are a. 1 only
correct? b. 2 only
a. 1 only c. Both 1 and 2
b. 2 only d. Neither 1 nor 2
c. Both 1 and 2 Answer: D
d. Neither 1 nor 2
Explanation:
Answer: A
Eco-Sensitive Zones are the areas that are
Explanation: Supreme Court in number of declared under the Environment (Protection)
cases have widened the scope of Article 21 to Act, 1986
include many natural rights in it. One such right
is Right to healthy Environment. Parliament of Activities permitted in this area: Rain water
India have enacted NGT Act 2010, in order to harvesting, organic farming, use of renewable
implement the provisions of this article. energy resources, agriculture and horticulture
practices by local communities, tourism.
Article 275(1) deals with the grants to the states
for the welfare of Scheduled Tribes and Restricted activities in this area: Felling of
strengthening of administration of tribal areas. trees, drastic change in agriculture systems,
commercial use of natural water resources (like
Educational Objective: to know the ground water harvesting), setting up hotels and
Environment laws in India resorts
Style: Which based Educational objective: to know about the major
Substance: Environmental laws eco sensitive zones, statutory provisions.

Form: Two statement question Style: reference based

8. With reference to 'Eco-Sensitive Zones', Substance: Environmental policies and laws


which of the following statements is/are Form: two statement question
correct? [2014]
4.1.2.2. Water Acts/Policies
1) Eco-Sensitive Zones are the areas that are
declared under the Wildlife (Protection) Act, 9. If National Water Mission is properly and
1972. completely implemented, how will it
2) The purpose of the declaration of Eco- impact the country? [2012]
Sensitive Zones is to prohibit all kinds of 1. Part of the water needs of urban areas will
human activities in those zones except be met through recycling of waste-water.
agriculture. 2. The water requirements of coastal cities
with inadequate alternative sources of water
will be met by adopting appropriate

www.laexias.com Page 53 https://elearn.laex.in


 
     
RRP 2022 - Environment UPSC
Previous Year Questions
               
technologies that allow for the use of ocean 6. Adopt large scale irrigation programmes
water which rely on sprinklers, drip irrigation and
3. All the rivers of Himalayan origin will be ridge and furrow irrigation.
linked to the rivers of peninsular India Educational objective: To know about impact
4. The expenses incurred by farmers for of National Water Mission.
digging bore- wells and for installing motors
and pump-sets to draw ground-water will be Style: Descriptive based
completely reimbursed by the Government
Substance: Environmental policies
Select the correct answer using the codes
Form: Four statement question
a. 1 only
4.1.3. Biodiversity conservation related laws
b. 1 and 2 only
4.1.3.1. Wildlife Protection laws
c. 3 and 4 only
d. 1, 2, 3 and 4 10. If a particular plant species is placed
Answer: B under Schedule VI of The Wildlife
Protection Act, 1972, what is the
Explanation: National Water Mission: implication? [2020]
It is one of the eight missions launched under a) A licence is required to cultivate that plant.
the National Action Plan on Climate Change b) Such a plant cannot be cultivated under
(NAPCC) for combating the threats of global any circumstances.
warming. c) It is a Genetically Modified crop plant.
d) Such a plant is invasive and harmful to the
Objectives of the National Water Mission are,
ecosystem.
1. increase water use efficiency by 20% Answer: A
through regulations, differential
entitlements and pricing. Explanation:
2. A considerable share of water needs of
Wildlife Protection Act, 1972 gives protection to
urban areas is to be met through recycling
wild animals, birds and plants
of wastewater.
3. Water requirements of coastal cities are to The species are placed under various schedules
be met through the adoption of low- ranging from Schedule I to schedule VI.
temperature desalination technologies. Schedule VI contains those species that are
4. Consult with states to ensure that basin- forbidden for cultivation. A license is required to
level management strategies are made to cultivate such plant.
deal with variability in rainfall and river
Educational Objective: To know the provisions
flows due to climate change.
of Wildlife Protection Act, 1972
5. Enhance storage above and below ground,
implement rainwater harvesting. Style: What based

Substance: Environmental laws

www.laexias.com Page 54 https://elearn.laex.in


 
     
RRP 2022 - Environment UPSC
Previous Year Questions
               
Form: single answered a. It enjoys the same level of protection as the
tiger.
11. According to the Wildlife (Protection)
b. It no longer exists in the wild, a few
Act, 1972, which of the following animals
individuals are under captive protection;
cannot be hunted by any person except
and now it is impossible to prevent its
under some provisions provided by law? [2017]
extinction.
1) Gharial c. It is endemic to a particular region of India.
2) Indian wild ass d. Both (b) and (c) stated above are correct in
3) Wild buffalo this context.
Select the correct answer using the code Answer: A
given below
Explanation: Wild Life Protection Act –
a. 1 only Schedules
b. 2 and 3 only
c. 1 and 3 only Tiger in India is placed under Schedule I of
d. 1,2 and 3 Wildlife (Protection) Act 1972.

Answer: D The act provides for six schedules to regulate the


trade on animals.
Explanation: All the three animals come under
Schedule I of Wildlife (Protection) Act 1972 as animals which are in the
endangered species. Schedule I category of endangered
and part II of species. These are given
Harming or hunting the species listed in
schedule II absolute protection from
Schedule I of the Act is prohibited throughout
hunting. Eg Tiger
India. Therefore, all these animals are given
absolute protection from hunting, except under These also have roughly the
some provisions provided by law. same provisions of Schedule I
Schedule III
and II, but cover animals that
Educational Objective: To have knowledge and IV
are not in danger of becoming
about the conservation status of various
extinct.
animals.
delineates animals that can be
Style: Reference based
hunted like ducks and deers
Substance: Environmental laws Schedule V with the prior permission of
chief wildlife warden.
Form: List based
(Vermin)
12. In India, if a species of tortoise is
concerns cultivation and plant
declared protected under Schedule I of
Schedule VI life and gives teeth to setting up
the Wildlife (Protection) Act, 1972, what
more protected national parks.
does it imply? [2017]

www.laexias.com Page 55 https://elearn.laex.in


 
     
RRP 2022 - Environment UPSC
Previous Year Questions
               
Educational Objective: to have idea about the not require a felling permission or transit
degree of protection that various animals enjoy permission from any state forest department.
under Wild Life Protection Act, 1972.
Section 2(i) of the Forest Rights Act defines a
Style: What based Minor Forest Produce (MFP) as all non-timber
forest produce of plant origin and includes
Substance: Environmental laws
bamboo, brushwood, stumps, canes, cocoon,
Form: single answered honey, waxes, Lac, tendu/kendu leaves,
medicinal plants etc.
4.1.3.2. Forest conservation laws
FRA Act, 2006 recognises community rights of
13. Consider the following statements: [2019]
forest dwellers which include, right to collect
1) As per recent amendment to the Indian and sell Minor Forest Produce, rights to pasture,
Forest Act, 1927, forest dwellers have the water bodies and diversion of up to one hectare
right to fell the bamboos grown on the forest (ha) forestland for community infrastructure like
areas. schools.
2) As per the Scheduled Tribes and Other
Traditional Forest Dwellers (Recognition of Educational Objective: to have knowledge
Forest Rights) Act, 2006, bamboo is a minor about the rights of tribes as per the forest
forest produce. dwellers rights Act
3) The Scheduled Tribes and Other Traditional Style: Consider based
Forest Dwellers (Recognition of Forest
Rights) Act, 2006 allows ownership of minor Substance: Environmental laws
forest produce to forest dwellers. Form: Three statement based
Which of the statements given above is/are
14. Consider the following statements: [2018]
correct?
1) The definition of "Critical Wildlife Habitat" is
a. 1 and 2 only incorporated in the Forest Rights Act, 2006.
b. 2 and 3 only 2) For the first time in India, Baigas have been
c. 3 only given Habitat Rights.
d. 1, 2 and 3 3) Union Ministry of Environment, Forest and
Answer: B Climate Change officially decides and
declares Habitat Rights for Primitive and
Explanation:
Vulnerable Tribal Groups in any part of
Parliament amended Section 2(7) of Indian India.
Forest Act, 1927. Which of the statements given above is/are
As per this amendment, bamboo is no longer a correct?
tree and felled bamboo too is not timber. So, any a. 1 and 2 only
bamboo grown in private or homestead land b. 2 and 3 only
(not forest) by farmers or private growers does c. 3 only
d. 1, 2 and 3
www.laexias.com Page 56 https://elearn.laex.in
 
     
RRP 2022 - Environment UPSC
Previous Year Questions
               
Answer: A 2) Launching the second green revolution to
enhance agricultural output so as to ensure
Explanation:
food security to one and all in the future.
Forest Rights Act, 2006 defined Critical Wildlife 3) Restoring and enhancing forest cover and
Habitats as areas of national parks and responding to climate change by a
sanctuaries that are established for wildlife combination of adaptation and mitigation
conservation and these areas should be kept as measures
inviolate (i.e., shouldn’t have any human Select the correct answer using the code
presence including forest dwellers). Therefore, given below.
statement 1 is correct.
a. 1 only
Baiga community is a Particularly Vulnerable b. 2 and 3 only
Tribal Group (PVTG) of Madhya Pradesh. c. 3 only
In a bid to undo historical injustice to Baigas, d. 1, 2 and 3
habitat rights under the Forest Rights Act of Answer: C
2006 were accorded to them. Therefore,
Explanation: National Mission for a Green India
Statement 2 is correct.
or the commonly called Green India Mission
The Ministry of Tribal Affairs is implementing (GIM), is one of the eight Missions outlined
the ―Scheme of Development of Particularly under India’s action plan for addressing the
Vulnerable Tribal Groups (PVTGs). It aims at challenge of climate change -the National Action
planning their socio-economic development in a Plan on Climate Change (NAPCC). GIM,
comprehensive manner while retaining the launched in February 2014, is aimed at
culture and heritage of the community. protecting, restoring and enhancing India’s
diminishing forest cover and responding to
Educational Objective: Forest Rights Act is
climate change by a combination of adaptation
one of the landmark legislations in India for
and mitigation measures. The mission has the
granting rights to the tribes.
broad objective of both increasing the forest and
Style: Consider based tree cover by 5 million ha, as well as increasing
the quality of the existing forest and tree cover in
Substance: Environmental laws and policies
another 5 million ha of forest/non forest lands
Form: Three statement question in 10 years. The Mission proposes a holistic view
of greening and focuses not on carbon
15. Which of the following best describes/
sequestration targets alone, but also, on
describe the aim of 'Green India Mission'
multiple ecosystem services, especially,
of the Government of India? [2016]
biodiversity, water, biomass etc., along with
1) Incorporating environmental benefits and provisioning services like fuel, fodder, timber
costs into the Union and State Budgets and non-timber forest produces. It will also
thereby implementing the 'green increase options of forest-based livelihood of
accounting'. households living in the fringe of those
landscapes where the Mission is implemented.

www.laexias.com Page 57 https://elearn.laex.in


 
     
RRP 2022 - Environment UPSC
Previous Year Questions
               
Green India mission is in no way associated with Allocation of these rights
Green accounting of Central and State budgets,
The Act gives authority to Gram Sabha to
nor has it got anything to do with the second
initiate the process for determining the nature
Green revolution. Hence both statements 1 and
and extent of individual or community forest
2 are incorrect.
rights or both to be given to the forest dwelling
Educational Objective: Green India mission Scheduled Tribes and other traditional forest
and its objectives. dwellers within the local limits of its jurisdiction.

Style: Which based Educational Objective: To know about Forest


Rights Act 2006
Substance: Environmental initiatives and
policies Style: Who based

Form: Three statement question Substance: Environmental laws

16. Under the Scheduled Tribes and Other Form: Single Answered
Traditional Forest Dwellers (Recognition
17. With reference to India, consider the
of Forest Rights) Act, 2006, who shall be
following Central Acts: [2011]
the authority to initiate the process for
determining the nature and extent of 1) Import and Export (Control) Act, 1947
individual or community forest rights or 2) Mining and Mineral Development
both? [2013] (Regulation) Act, 1957
3) Customs Act, 1962
a. State Forest Department
4) Indian Forest Act, 1927
b. District Collector/Deputy Commissioner
c. Tahsildar/Block Development Officer Which of the above Acts have relevance
/Mandal Revenue Officer to/bearing on the biodiversity conservation
d. Gram Sabha in the country?

Answer: D a. 1 and 3 only


b. 2, 3 and 4 only
Explanation: Forest Rights Act, 2006
c. 1, 2, 3 and 4
The act recognizes and vest the forest rights and d. None of the above Acts
occupation in Forest land in forest Dwelling Answer: C
Scheduled Tribes (FDST) and Other Traditional
Forest Dwellers (OTFD) who have been residing Explanation:
in such forests for generations
Totally there 35 Acts that are legislated by the
It identifies 4 rights Indian Parliament that deal with Biodiversity in
one or the other way. All the above 4 Acts Deal
1) Title rights with the conservation of Biodiversity.
2) Use rights
3) Relief and development rights While Option 2 and 4 are very certain to deal
4) Forest management rights with the biodiversity, the other two options also

www.laexias.com Page 58 https://elearn.laex.in


 
     
RRP 2022 - Environment UPSC
Previous Year Questions
               
deal with Biodiversity indirectly. Ex: both the 40% of the world’s energy consumption and
acts deal with entering of Alien Species related 1/3rd of global greenhouse gas (GHG)
(Invasive Species) from other countries. emissions.

Educational Objective: to have knowledge The building sector has more potential to deliver
about various Environmental Legislations quick, deep and cost-effective GHG mitigation
than any other.
Style: Reference based
Significantly increasing building energy
Substance: Environmental Acts
efficiency can be achieved in the short term.
Form: List based
Energy consumption in both new and existing
4.1.4. International Agreements/Conventions buildings could be cut by an estimated 30-50%
4.1.4.1. Climate change by 2020 through readily available technologies,
design, equipment, management systems, and
18. The “Common Carbon Metric” supported
alternative generation solutions.
by UNEP, has been developed for (2021)
This can be funded through investments that
(a) Assessing the carbon footprint of building
quickly pay back and result in significant
operations around the world
environmental, social, and economic benefits.
(b) Enabling commercial farming entities
around the world to enter carbon emission A universal measuring stick for building
trading emissions a Common Carbon Metric provides
(c) Enabling governments to assess the overall the foundation for accurate performance
carbon footprint caused by their countries baselines to be drawn, national targets set, and
(d) Assessing the overall carbon foot-print carbon traded on a level playing field.
caused by the use of fossil fuels by the
world in a unit time While all stages of a building’s life-cycle
(including construction and demolition) produce
Answer: A
carbon emissions, the building’s operational
Explanation: phase accounts for 80-90% of emissions
resulting from energy use mainly for heating,
The Common Carbon Metric is the calculation cooling, ventilation, lighting and appliances.
used to define the measurement, reporting, and
verification for GHG emissions associated with Therefore, this is the stage of the building’s life-
the operation of buildings types of particular cycle that is the focus of the Common Carbon
climate regions. Metric.

Common Carbon Metric provides the foundation Educational objective: Common Carbon Metric
for accurate performance baselines to be drawn, has been developed for Assessing the carbon
national targets set, and carbon traded on a footprint of building operations around the
level playing field. world.

The purpose of a Common Carbon Metric for Form: Single statement


buildings is to give the sector that represents
www.laexias.com Page 59 https://elearn.laex.in
 
     
RRP 2022 - Environment UPSC
Previous Year Questions
               
Style: Description based question deforestation and forest degradation as part of a
post-2020 global climate agreement.
Substance: Environmental laws, policies,
initatives. Statement 3 is incorrect. The New York
Declaration on Forests (NYDF) is a voluntary
19. With reference to the ‘’New York
and non-binding international declaration to
Declaration on Forests’’, which of the
take action to halt global deforestation.
following statements are correct? (2021)
1. It was first endorsed at the United Nations Statement 4 is correct. It has presently over 200
Climate Summit in 2014 endorsers – including national governments,
2. It endorses a global timeline to end the loss subnational governments, companies,
of forests indigenous groups, financial institutions and
3. It is a legally binding international NGOs. These endorsers have committed to doing
declaration their part to achieve the NYDF goals and follow
4. It is endorsed by governments, big its accompanying action agenda.
companies and indigenous communities. Statement 5 is incorrect. Presently, India has
5. India was one of the signatories at its not signed the New York Declaration on Forests
inception (NYDF).
Select the correct answer using the code
Educational objective: “New York Declaration
given below
on Forests” first endorsed at the United Nations
(a) 1, 2 and 4 Climate Summit in 2014, it endorses a global
(b) 1, 3 and 5 timeline to end the loss of forests.
(c) 3 and 4
Form: Five statements
(d) 2 and 5
Style: Reference Based
Answer: A
Substance: Environmental institutions based –
Explanation:
Global and International; - reports. Conferences
Statement 1 is correct. The New York and conventions
Declaration on Forests (NYDF) was endorsed at
20. In the context of India’s preparation for
the United Nations Climate Summit in
Climate-smart Agriculture, consider the
September 2014.
following statements: (2021)
Statement 2 is correct. New York Declaration on
1. The ‘Climate-Smart village’ approach in
Forests (NYDF) emphasises ten goals which
India is a part of a project led by climate
includes halting natural forest loss by 2030. The
change, Agriculture and food security
targets also include restoring 350 million
(CCAFS), an international research
hectares of degraded landscapes and
programme.
forestlands, improving governance, increasing
2. The project of CCAFS is carried out under
forest finance, and reducing emissions from
Consultative Group on International

www.laexias.com Page 60 https://elearn.laex.in


 
     
RRP 2022 - Environment UPSC
Previous Year Questions
               
Agricultural Research (CGIAR) Saharan Africa with a wide array of partners
headquartered in France. throughout the world.
3. The International Crops Research Institute
Educational objective: ICRISAT in India is one
for the Semi-Arid Tropics (ICRISAT) in India
of the CGIAR’S research centers, Climate-Smart
is one of the CGIAR’S research centers.
village’ approach in India is a part of a project
Which of the statements given above are led by CCAFS.
correct?
Form: Three statement Based question
(a) 1 and 2 only
(b) 2 and 3 only Style: Reference Based
(c) 1 and 3 only Substance: Environmental institutions based –
(d) 1,2 and 3 Global and International; - reports. Conferences
Answer: D and conventions

Explanation: 21. With reference to the ‘Global Alliance for


Climate-Smart Agriculture (GACSA)’,
Statement 1 is correct. The Climate-Smart
which of the following statements is/are
Village project in India is a program of CGIAR
correct? [2018]
Research Program on Climate Change,
Agriculture and Food Security (CCAFS). The 1) GACSA is an outcome of the Climate
CCAFS started piloting the Climate-Smart Summit held in Paris in 2015.
Village in 2012 in Africa (Burkina Faso, Ghana, 2) Membership of GACSA does not create any
Mali, Niger, Senegal, Kenya, Ethiopia, Tanzania, binding obligations.
and Uganda) and South Asia (Bangladesh, India, 3) India was instrumental in the creation of
and Nepal). GACSA.
Select the correct answer using the code
Statement 2 is correct. Climate Change,
given below:
Agriculture and Food Security (CCFAS) is carried
out under CGIAR (formerly the Consultative a. 1 and 3 only
Group for International Agricultural Research). b. 2 only
Headquarter of CGIAR is in Montpellier, France. c. 2 and 3 only
CGIAR is a global partnership that unites d. 1, 2 and 3
international organizations engaged in research
Answer: B
about food security.
Explanation:
Statement 3 is correct. International Crops
Research Institute for the Semi-Arid Tropics The ‘Global Alliance for Climate-Smart
(ICRISAT) is a CGIAR Research Center. ICRISAT Agriculture’ was launched during the Climate
is a non-profit, non-political public international Summit, on 23 September, 2014.
research organization that conducts agricultural
The Global Alliance for Climate-Smart
research for development in Asia and sub-
Agriculture (CSA), will work for: sustainable and
equitable increases in agricultural productivity

www.laexias.com Page 61 https://elearn.laex.in


 
     
RRP 2022 - Environment UPSC
Previous Year Questions
               
and incomes; increased resilience of food Explanation: The Global Climate Change
systems and farming livelihoods; and reduced Alliance (GCCA) is a European Union flagship
greenhouse gas (GHG) emissions from initiative which is helping the world's most
agriculture. The Alliance will also aim to boost vulnerable countries to address climate change.
food and nutrition security through climate- Having started with just four pilot projects in
adjusted and natural-resource efficient 2008, it has become a major climate initiative
agricultural practices, food systems and social that has funded over 80 projects of national,
policies. It is evident that the alliance does not regional and worldwide scope in Africa, Asia, the
impose any binding obligation on member Caribbean and the Pacific. Neither World
countries. Resources Institute (WRI) nor World Business
Council for Sustainable Development (WBCSD)
The alliance is an initiative of the Food and
has anything to do with GCCA.
Agriculture Organisation (FAO).
Educational Objective: Various international
Educational Objective: Various initiatives
Organisations and their mandate.
taken towards mitigating the impact of climate
change. Style: reference based

Style: Reference based Substance: International initiative

Substance: Global initiative Form: Three statement question

Form: Three statement question 23. Consider the following statements: [2017]

22. With reference to ‘Global Climate Change 1) Climate and Clean Air Coalition (CCAC) to
Alliance’, which of the following Reduce Short Lived Climate Pollutants is a
statements is/are correct? [2017] unique initiative of G20 group of countries.
2) The CCAC focuses on methane, black
1. It is an initiative of the European Union.
carbon and hydrofluorocarbons.
2. It provides technical and financial support
to targeted developing countries to integrate Which of the statements given above is/are
climate change into their development correct?
policies and budgets. a. 1 only
3. It is coordinated by World Resources b. 2 only
Institute (WRI) and World Business Council c. Both 1 and 2
for Sustainable Development (WBCSD). d. Neither 1 nor 2
Select the correct answer using the code Answer: B
given below:
Explanation: The Climate & Clean Air Coalition
a. 1 and 2 only is the only global effort that unites governments,
b. 3 only civil society and private sector, committed to
c. 2 and 3 only improving air quality and protecting the climate
d. 1, 2 and 3 in next few decades by reducing short-lived
Answer: A climate pollutants across sectors. It is an

www.laexias.com Page 62 https://elearn.laex.in


 
     
RRP 2022 - Environment UPSC
Previous Year Questions
               
initiative of the United Nations Environment ratification by all the member countries and was
Program and not G20. scheduled to get into effect from 2020. The Paris
Agreement's long-term temperature goal is to
The Coalition's initial focus is on methane, black
keep the increase in global average temperature
carbon, and HFCs.
to well below 2 °C above pre-industrial levels;
Educational Objective: Various initiatives and to pursue efforts to limit the increase to 1.5
started towards combating GHG emissions. °C, recognizing that this would substantially
reduce the risks and impacts of climate change.
Style: Consider based
The Paris Agreement requires developed
Substance: Global Environmental initiative
countries to raise finances with $100 billion per
Form: Two statement question year as the floor by 2020, to help developing
nations in both mitigation and adaptation
24. With reference to the Agreement at the activities, while other nations are encouraged to
UNFCCC Meeting in Paris in 2015, which provide funding voluntarily.
of the following statements is/are
correct? [2016] Educational Objective: Learn about important
climate change related agreements.
1) The Agreement was signed by all the
member countries of the UN and it will go Style: reference based
into effect in 2017
Substance: International conventions
2) The Agreement aims to limit the greenhouse
gas emissions so that the rise in average Form: Three statement question
global temperature by the end of this
century does not exceed 2°C or even 1.5°C 25. The term ‘Intended Nationally
above pre-industrial levels Determined Contribution’ is sometimes
3) Developed countries acknowledged their seen in the news in the context of: [2016]
historical responsibility in global warming a. Pledges made by the European countries to
and committed to donate $ 1000 billion a rehabilitate refugees from the war-affected
year from 2020 to help developing countries Middle East
to cope with climate change. b. Plan of action outlined by the countries of
Select the correct answer using the code the world to combat climate change
given below. c. Capital contributed by the member
countries in the establishment of Asian
a. 1 and 3 only Infrastructure Investment Bank
b. 2 only d. Plan of action outlined by the countries of
c. 2 and 3 only the world regarding Sustainable
d. 1, 2 and 3 Development Goals.
Answer: B Answer: B

Explanation: Though the agreement was Explanation: Countries across the globe
reached on consensus, it is subject to adopted an historic international climate

www.laexias.com Page 63 https://elearn.laex.in


 
     
RRP 2022 - Environment UPSC
Previous Year Questions
               
agreement at the U.N. Framework Convention on Clean Development Mechanism (CDM) allows an
Climate Change (UNFCCC) Conference of the industrialized/developed country with an
Parties (COP21) in Paris in December 2015. In emission-reduction or emission-limitation
anticipation of this moment, countries publicly commitment under the Kyoto Protocol (called as
outlined what post-2020 climate actions they Annex I Party or Annex B Party of the original
intended to take under the new international Kyoto Protocol signed in 1997) to implement an
agreement, known as their Intended Nationally emission-reduction project in any of those
Determined Contributions (INDCs). developing countries (which may otherwise be
not financially capable of undertaking such
Educational Objective: To know about Paris
projects), thereby earning them tradable
agreement
Certified Emission Reduction (CER) credits, each
Style: Current Affairs equivalent to one tonne of CO2. The saleable
CERs earned from such projects can be counted
Substance: Terminology based
towards meeting the prescribed Kyoto targets.
Form: Single answered
Cartagena and Nagoya Protocols are related to
26. Consider the following pairs: [2016] convention on biodiversity

Terms Their Origin Educational Objective: Students' familiarity


with terms associated with International
1) Annex-1 : Cartagena
protocols and treaties.
Countries Protocol
Style: consider based
2) Certified : Nagoya Protocol
Emissions Reductions Substance: Terminology based

3) Clean : Kyoto Protocol Form: matching based


Development Mechanism
27. What is 'Greenhouse Gas Protocol’? [2016]
Which of the pairs given above is/ are
a. It is an international accounting tool for
correctly matched?
government and business leaders to
a. 1 and 2 only understand, quantify and manage
b. 2 and 3 only greenhouse gas emissions
c. 3 only b. It is an initiative of the United Nations to
d. 1, 2 and 3 offer financial incentives to developing
countries to reduce greenhouse gas
Answer: C
emissions and to adopt eco-friendly
Explanation: technologies
c. It is an inter-governmental agreement
Annex I, Certified Emissions Reductions, Clean
ratified by all the member countries of the
Development Mechanism all are associated with
United Nations to reduce greenhouse gas
Kyoto Protocol itself.
emissions to specified levels by the year
2022

www.laexias.com Page 64 https://elearn.laex.in


 
     
RRP 2022 - Environment UPSC
Previous Year Questions
               
d. It is one of the multilateral REDD+ Answer: A
initiatives hosted by the World Bank.
Explanation:
Answer: A
The Green Climate Fund is a fund established
Explanation: within the framework of the UNFCCC as a
GHG Protocol establishes comprehensive global mechanism to redistribute money from
standardized frameworks to measure and developed to the developing world in adaptation
manage greenhouse gas (GHG) emissions from and mitigation practices to counter climate
private and public sector operations, value change.
chains and mitigation actions. GCF launched its initial resource mobilization in
GHG Protocol supplies the world's most widely 2014, and rapidly gathered pledges worth USD
used greenhouse gas accounting standards. The 10.3 billion. These funds come mainly from
Corporate Accounting and Reporting Standard developed countries, but also from some
provides the accounting platform for virtually developing countries, regions.
every corporate GHG reporting program in the Educational Objective: Climate funds, their
world. mandate and source of funding.
Educational Objective: Test candidate's Style: Which based
understanding of Greenhouse gas protocol.
Substance: Global Environmental Institution
Style: What based question
Form: Two statement
Substance: Global Environmental initiative
29. Consider the following statements
Form: Single answered regarding 'Earth Hour': [2014]
28. Which of the following statements 1. It is an initiative of UNEP and UNESCO.
regarding 'Green Climate Fund' is/are 2. It is a movement in which the participants
correct? [2015] switch off the lights for one hour on a
1) It is intended to assist the developing certain day every year.
countries in adaptation and mitigation 3. It is a movement to raise the awareness
practices to counter climate change. about the climate change and the need to
2) It is founded under the aegis of UNEP, save the planet.
OECD, Asian Development Bank and World Which of the statements given above is/ are
Bank. correct?
Select the correct answer using the code a. 1 and 3 only
given below. b. 2 only
a. 1 only c. 2 and 3 only
b. 2 only d. 1, 2 and 3
c. Both 1 and 2 Answer: C
d. Neither 1 nor 2

www.laexias.com Page 65 https://elearn.laex.in


 
     
RRP 2022 - Environment UPSC
Previous Year Questions
               
Explanation: is being replaced by the Paris Agreement in
2020.
Earth Hour is a worldwide movement for the
planet organized by the World-Wide Fund for One carbon credit is equal to one tonne of
Nature (WWF). The event is held worldwide carbon dioxide, or in some markets, carbon
annually encouraging individuals, communities, dioxide equivalent gases. Carbon trading is an
households and businesses to turn off their non- application of an emissions trading approach.
essential lights for one hour, from 8:30 to 9:30 Greenhouse gas emissions are capped and then
p.m. to raise the awareness about the climate markets are used to allocate the emissions
change and the need to save the planet. among the group of regulated sources.

Educational objective: to know about the The goal is to allow market mechanisms to drive
international environmental initiatives. industrial and commercial processes in the
direction of low emissions or less carbon
Style: Consider based
intensive approaches than those used when
Substance: Global Environmental Initiative there is no cost to emitting carbon dioxide and
other GHGs into the atmosphere.
Form: Three statement question
Market mechanisms determines the cost of each
30. Regarding "carbon credits", which one of
carbon credit and it is not fixed by UNEP.
the following statements is not correct?
[2011] Educational Objective: Understand various
terminologies that are in news.
a. The carbon credit system was ratified in
conjunction with the Kyoto Protocol Style: reference based
b. Carbon credits are awarded to countries or
groups that have reduced greenhouse gases Substance: Terminology based
below their emission quota Form: Single answered
c. The goal of the carbon credit system is to
4.1.4.2. Desertification
limit the increase of carbon dioxide
emission 31. What is / are the importance/
d. Carbon credits are traded at a price fixed importance’s of the 'United Nations
from time to time by the United Nations Convention to Combat Desertification'?
Environment Programme. [2016]
Answer: D 1. It aims to promote effective action through
innovative national programmes and
Explanation:
supportive international partnerships.
A carbon credit represents the right to emit a 2. It has a special/particular focus on South
measured amount of GHG. Carbon credits work Asia and North Africa regions, and its
as a certification that business or individual Secretariat facilitates the allocation of major
owning them is counterbalancing the emission portion of financial resources to these
of greenhouse gases (GHG). Carbon credit and regions.
markets existed under the Kyoto Protocol, which
www.laexias.com Page 66 https://elearn.laex.in
 
     
RRP 2022 - Environment UPSC
Previous Year Questions
               
3. It is committed to bottom-up approach,
encouraging the participation of local people 4.1.4.3. Biodiversity loss
in combating the desertification.
32. Consider the following statements [2019]
Select the correct answer using the code
given below. 1. Under Ramsar convention, it is mandatory
on the part of the Government of India to
a. 1 only
protect and conserve all the wetlands in the
b. 2 and 3 only
territory of India.
c. 1 and 3 only
2. The Wetlands (Conservation and
d. 1, 2 and 3
Management) Rules, 2010 were framed by
Answer: C the Government of India based on the
recommendations of Ramsar convention.
Explanation: Established in 1994, the United
3. The Wetlands (Conservation and
Nations Convention to Combat Desertification
Management) Rules, 2010 also encompass
(UNCCD) is the sole legally binding international
the drainage area or catchment regions of
agreement linking environment and development
the wetlands as determined by the
to sustainable land management. The
authority.
Convention addresses specifically the arid, semi-
arid and dry sub-humid areas, known as the Which of the statements given above is/are
drylands, where some of the most vulnerable correct?
ecosystems and peoples can be found.
a) 1 and 2 only
The Convention is committed to bottom-up / b) 2 and 3 only
Decentralized approach, encouraging the c) 3 only
participation of local people in combating the d) 1,2 and 3
desertification. Answer: C
The secretariat helps in compiling information Explanation: The Convention on Wetlands
and reports, not in allocating finances and the [waterfowl convention/Ramsar Convention] is an
south Asian region is not in focus. intergovernmental treaty that provides the
Educational objective: To know about UNCCD. framework for national action and international
cooperation for the conservation and wise use of
Style: What based wetlands and their resources

Substance: Global convention The government has to protect only the


recognised wetlands of international
Form: Three statement question
importance and is voluntary on part of
government with respect to other wetlands.

The Wetlands (Conservation and Management)


Rules, 2010 are made independent of Ramsar
convention for regulating various activities with

www.laexias.com Page 67 https://elearn.laex.in


 
     
RRP 2022 - Environment UPSC
Previous Year Questions
               
respect to all wetlands in the country by advocacy and engagement with governments
encompassing the drainage and catchment areas corporates and various conventions.
of wetlands.
Educational Objective: To know about various
Educational Objective: To learn related rules organizations associated with wetland protection
and conventions of wetlands. and management.

Style: Consider based Style: Reference based

Substance: International environmental Substance: International environmental


convention initiatives

Form: Three statement question Form: Two statement question

33. With reference to a conservation 34. If a wetland of international importance


organization called ‘Wetlands is brought under the 'Montreux Record',
International’, which of the following what does it imply? [2019]
statements Is/are correct? [2019] a. Changes in ecological character have
1. It is an intergovernmental organization occurred, are occurring or are likely to
formed by the countries which are occur in the wetland as a result of human
signatories to Ramsar Convention. interference
2. It works at the field level to develop and b. The country in which the wetland is located
mobilize knowledge, and use the practical should enact a law to prohibit any human
experience to advocate for better policies. activity within five kilometres from the edge
Select the correct answer using the codes of the wetland
given below. c. The survival of the wetland depends on the
cultural practices and traditions of certain
a) 1 only communities living in its vicinity and
b) 2 only therefore the cultural diversity therein
c) Both 1 and 2 should not be destroyed
d) Neither 1 nor 2 d. It is given the status of 'World Heritage Site'
Answer: B Answer: A

Explanation: Explanation:

Wetlands International is a global organisation Montreux Record is the principal tool under the
that works to sustain and restore wetlands and Ramsar convention for highlighting wetlands
their resources for people and biodiversity. sites of national importance that are adversely
Wetlands International is an Independent not- changing their ecological character due to
for-profit organization working in the field of technological developments, pollution or
wetland conservation. other human interference and are in need of
priority conservation attention.
Wetlands International’s work ranges from
research to community-based field projects to

www.laexias.com Page 68 https://elearn.laex.in


 
     
RRP 2022 - Environment UPSC
Previous Year Questions
               
Wetlands from India in Montreux record: Loktak conservation of all the sites through an
lake and Keoladeo national park ecosystem approach and allows their
simultaneous and sustainable use.
Educational Objective: To know the role of
Montreux record in Wetland Conservation Educational Objective: To learn about
International Wetland Conservation measures
Style: Descriptive
Style: Descriptive
Substance: International environmental
convention Substance: International environmental
convention
Form: Single answered
Form: Single answered
35. India is a party to the Ramsar
Convention and has declared many areas 36. Consider the following statements in
as Ramsar Sites. Which of the following respect of Trade Related Analysis of
statements best describes as to how we Fauna and Flora in Commerce (TRAFFIC):
should maintain these sites in the [2017]
context of this Convention? [2019] 1. TRAFFIC is a bureau under United Nations
a. Keep all the sites completely inaccessible to - Environment Programme (UNEP)
man so that they will not be exploited 2. The mission of TRAFFIC is to ensure that
b. Conserve all the sites through ecosystem trade in wild plants and animals is not a
approach and permit tourism and threat to the conservation of nature.
recreation only Which of the above statements is/are
c. Conserve all the sites through ecosystem correct?
approach for a period without any
exploitation, with specific criteria and a. 1 only
specific period for each site, and then allow b. 2 only
sustainable use of them by future c. Both 1 and 2
generations d. Neither 1 nor 2
d. Conserve all the sites through ecosystem Answer: B
approach and allow their simultaneous
sustainable use. Explanation: It is a NGO, not a bureau under
UNEP. TRAFFIC is governed by the TRAFFIC
Answer: D
Committee, a steering group composed of
Explanation: members of TRAFFIC’s partner organizations,
WWF and IUCN. It is working globally as a
The Convention on Wetlands [waterfowl Wildlife trade monitoring network.
convention/Ramsar Convention] is an
intergovernmental treaty that provides the It specializes in investigating and analysing
framework for national action and international wildlife trade trends, patterns, impacts and
cooperation for the conservation and wise use of drivers to provide the leading knowledge base on
wetlands and their resources. It encourages the trade in wild animals and plants.

www.laexias.com Page 69 https://elearn.laex.in


 
     
RRP 2022 - Environment UPSC
Previous Year Questions
               
Educational objective: to know about the benefits provided by ecosystems and
international environmental initiatives biodiversity, demonstrate their values in
economic terms and, where appropriate, capture
Style: Consider based
those values in decision-making
Substance: International environmental
Educational objective: to know about the
initiatives
international environmental initiatives
Form: Two statement question
Style: Reference based
37. With reference to an initiative called 'The
Substance: International environmental
Economics of Ecosystems and
initiatives
Biodiversity (TEEB)" which of the
following statements is/are correct? Form: Three statement question
[2016]
38. With reference to the International Union
1. It is an initiative hosted by UNEP, IMF and for Conservation of Nature and Natural
World Economic Forum. Resources (IUCN) and the Convention on
2. It is a global initiative that focuses on International Trade in Endangered
drawing attention to the economic benefits Species of Wild Fauna and Flora (Cities),
of biodiversity. which of the following statements is/are
3. It presents an approach that can help correct? [2015]
decision-makers recognize, demonstrate
1) IUCN is an organ of the United Nations and
and capture the value of ecosystems and
CITES is an international agreement
biodiversity
between governments
Select the correct answer using the code 2) IUCN runs thousands of field projects
given below, around the world to better manage natural
a. 1 and 2 only environments.
b. 3 only 3) CITES is legally binding on the States that
c. 2 and 3 only have joined it, but this Convention does not
d. 1, 2 and 3 take the place of national laws.

Answer: C Select the correct answer using the code


given below.
Explanation:
a. 1 only
The Economics of Ecosystems and Biodiversity b. 2 and 3 only
(TEEB) is a global initiative focused on “making c. 1 and 3 only
nature’s values visible”. Its principal objective is d. 1,2 and 3 only
to mainstream the values of biodiversity and
Answer: B
ecosystem services into decision-making at all
levels. It aims to achieve this goal by following a Explanation: IUCN is a voluntary organization,
structured approach to valuation that helps not an agency of United Nations. Some facts
decision-makers recognize the wide range of about IUCN:

www.laexias.com Page 70 https://elearn.laex.in


 
     
RRP 2022 - Environment UPSC
Previous Year Questions
               
Founded in 1948 as the world’s first global ‘United Nations Framework Convention on
environmental organisation, Today it is the Climate Change’.
largest professional global conservation network b. It undertakes scientific research on
environmental issues at global level
A leading authority on the environment and
c. It is an agency under OECD to facilitate the
sustainable development with more than 1,200
transfer of technology and funds to
member organizations including 200+
underdeveloped countries with specific aim
government and 900+ non-government
to protect their environment
organizations
d. Both a and b
A neutral forum for governments, NGOs,
scientists, business and local communities to Answer: D
find practical solutions to conservation and Explanation: Global Environment Facility, an
development challenges with thousands of field independently operating financial organization.
projects and activities around the world The GEF provides grants for projects related to
CITES was drafted as a result of a resolution biodiversity, climate change, international
adopted in 1963 at a meeting of members of the waters, land degradation, the ozone layer,
International Union for Conservation of Nature persistent organic pollutants (POPs), mercury,
(IUCN). States (countries) adhere voluntarily to sustainable forest management, food security,
the agreement are known as Parties. sustainable cities.

Although CITES is legally binding on the Parties The GEF also serves as financial mechanism
– in other words they have to implement the for the following conventions:
Convention – it does not take the place of 1. Convention on Biological Diversity (CBD)
national laws. Rather it provides a framework to 2. United Nations Framework Convention on
be respected by each Party, which has to adopt Climate Change (UNFCCC)
its own domestic legislation to ensure that 3. United Nations Convention to Combat
CITES is implemented at the national level. Desertification (UNCCD)
Educational objective: to know about the 4. Stockholm Convention on Persistent
international organizations. Organic Pollutants (POPs)
5. Minamata Convention on Mercury
Style: Reference based
The GEF, although not linked formally to the
Substance: International Environmental Montreal Protocol on Substances that Deplete
Initiatives the Ozone Layer (MP), supports implementation
of the Protocol in countries with economies in
Form: Three statement question
transition.
39. With reference to 'Global Environment
Educational objective: Understand the funding
Facility', which of the following
mechanism of various international bodies.
statements is/are correct? [2014]
a. It serves as financial mechanism for Style: Reference based
'Convention on Biological Diversity' and Substance: Global Environmental Institution

www.laexias.com Page 71 https://elearn.laex.in


 
     
RRP 2022 - Environment UPSC
Previous Year Questions
               
Form: Single answered question The International Treaty on Plant Genetic
Resources for Food and Agriculture aims at
40. Consider the following international
agreements: [2014] o recognizing the enormous contribution of

1. The International Treaty on Plant Genetic farmers to the diversity of crops that feed
Resources for Food and Agriculture the world.
o Access and benefit sharing
2. The United Nations Convention to Combat
Desertification Educational objective: To know about various
3. The World Heritage Convention International Agreements.
Which of the above has / have a bearing on Style: Consider based
the biodiversity?
Substance: Global Environmental Initiatives
a. 1 and 2 only
b. 3 only Form: List based
c. 1 and 3 only 41. The "Red Data Books" published by the
d. 1, 2 and 3 International Union for Conservation of
Answer: D Nature and Natural Resources (IUCN)
contain lists of [2011]
Explanation:
1) Endemic plant and animal species present
One of the functions of The World Heritage in the biodiversity hotspots.
Convention is to contain the most important 2) Threatened plant and animal species.
and significant natural habitats for in-situ 3) Protected sites for conservation of nature
conservation of biological diversity, including and natural resources in various countries.
those containing threatened species of
Select the correct answer using the codes
outstanding universal value from the point of
given below:
view of science or conservation. Created in
1972, the primary mission of the Convention is a. 1 and 3
to identify and protect the world's natural and b. 2 only
cultural heritage considered to be of c. 2 and 3
Outstanding Universal Value. d. 3 only

Established in 1994, the United Nations Answer: B


Convention to Combat Desertification
Explanation:
(UNCCD) is the sole legally binding international
agreement linking environment and development The IUCN publishes Red List/ red data book of
to sustainable land management. The Threatened Species, founded in 1964, is the
Convention addresses specifically the arid, semi- world’s most comprehensive inventory of the
arid and dry sub-humid areas, known as the global conservation status of biological species.
drylands
Species are classified by the IUCN Red List into
nine groups

www.laexias.com Page 72 https://elearn.laex.in


 
     
RRP 2022 - Environment UPSC
Previous Year Questions
               
1. Extinct (EX). Answer: D

2. Extinct in the wild (EW) Explanation:

3. Critically endangered (CR) Reducing emissions from deforestation and


forest degradation (REDD+) is a mechanism
4. Endangered (EN)
developed by Parties to the United Nations
5. Vulnerable (VU) Framework Convention on Climate Change
(UNFCCC). Since its formation in 2006, REDD+
6. Near threatened (NT)
had emerged as the most prominent global
7. Least concern (LC) mechanism to integrate the role of forests in
climate change. It was touted as a win-win
8. Data deficient (DD) situation for biodiversity conservation, carbon
9. Not evaluated (NE) sequestration and local livelihoods.

Educational objective: to know about IUCN’s It creates a financial value for the carbon stored
Red data Book. in forests by offering incentives for developing
countries to reduce emissions from forested
Style: Descriptive lands and invest in low-carbon paths to
Substance: International Environmental sustainable development.
Initiatives Developing countries would receive results-
Form: List based based payments for results-based actions.
REDD+ goes beyond simply deforestation and
4.1.4.4. Forest Management
forest degradation and includes the role of
42. Proper design and effective conservation, sustainable management of forests
implementation of UN REDD+ Programme and enhancement of forest carbon stocks. Hence
can significantly contribute to [2016] it covers all the options mentioned in the
question.
1) Protection of biodiversity
2) Resilience of forest ecosystems Educational Objective: Learn about UNFCCC’s
3) Poverty reduction initiative UN-REDD+.
Select the correct answer using the code Style: Descriptive
given below.
Substance: Global environmental convention
a. 1 and 2 only
b. 3 only Form: List based
c. 2 and 3 only
d. 1, 2 and 3

www.laexias.com Page 73 https://elearn.laex.in


 
     
RRP 2022 - Environment UPSC
Previous Year Questions
               
43. With reference to 'Forest Carbon with 47 developing countries across the world
Partnership Facility', which of the and 17 donors.
following statements is/are correct?
Educational Objective: Various initiatives
[2015]
under REDD+.
1) It is a global partnership of governments,
businesses, civil society and indigenous Style: Reference based
peoples. Substance: Global environmental Initiative
2) It provides financial aid to universities,
individual scientists and institutions Form: Three statement question
involved in scientific forestry research to 1. Environment Related Institutions
develop eco-friendly and climate adaptation
and Organisations
technologies for sustainable forest
management. 4.2.1. National
3) It assists the countries in their 'REDD+ 4.2.1.1. Pollution related agencies
(Reducing Emissions from Deforestation
44. Consider the following statements: [2019]
and Forest Degradation+)' efforts by
providing them with financial and technical 1) Animal Welfare Board of India is established
assistance. under the Environment (Protection) Act,
1986.
Select the correct answer using the code
2) National Tiger Conservation Authority is a
given below.
statutory body.
a. 1 only 3) National Ganga River Basin Authority is
b. 2 and 3 only chaired by the Prime Minister.
c. 1 and 3 only Which of the statements given above is / are
d. 1, 2 and 3 correct?
Answer: C
a. 1 only
Explanation: b. 2 and 3 only
c. 2 only
The Forest Carbon Partnership Facility (FCPF) is
d. 1, 2 and 3
a World Bank multi-donor fund of governments
and non-governmental entities, including private Answer: B
companies, and consists of two separate but Animal Welfare Board of India
complementary funding mechanisms, namely a
Readiness Fund and a Carbon Fund. The FCPF • It is a statutory body, stablished in 1962
was created to assist developing countries to under Section 4 of The Prevention of Cruelty
reduce emissions from deforestation and forest to Animals Act,1960 under Ministry of
degradation, enhance and conserve forest Environment and Forests
carbon stocks, and sustainably manage forests • It is an advisory body and frames a range of
(REDD+). Launched in 2008, the FCPF works rules on how animals ought to be humanely
treated everywhere.

www.laexias.com Page 74 https://elearn.laex.in


 
     
RRP 2022 - Environment UPSC
Previous Year Questions
               
National Ganga River Basin Authority c. Both 1 and 2
• Constituted under Section 3(3) of the d. Neither 1 nor 2
Environment (Protection) Act, 1986 Answer: B
• The Ministry of Water Resources, River
Development and Ganga Rejuvenation Explanation:
(MoWR, RD & GR) is the nodal Ministry for
Both NGT and CPCB are statutory bodies i.e.,
the NGRBA
established by acts
• Chaired by the Prime Minister and has as
its members the Union Ministers concerned, NGT is established by NGT ACT itself where as
the Chief Ministers of the States through CPCB, is statutory organisation constituted in
which Ganga flows. 1974 under the Water (Prevention and Control of
Pollution) Act, 1974. Hence option 1 is wrong
National Tiger Conservation Authority
The National Green Tribunal is dedicated
• It is a statutory body under the Ministry of
jurisdiction in environmental matters and
Environment, Forests and Climate Change
provides speedy environmental justice and helps
constituted under enabling provisions of the
reduce the burden of litigation in the higher
Wildlife (Protection) Act, 197.
courts.
Educational Objective: to have knowledge
about various environmental organisations Principal functions of the CPCB, as spelt out in
the Water (Prevention and Control of Pollution)
Style: Consider based Act, 1974, and the Air (Prevention and Control of
Substance: Environmental Laws Pollution) Act, 1981, to

Form: List based (i) Promote cleanliness of streams and wells in


different areas of the States by prevention,
45. How is the National Green Tribunal (NGT) control and abatement of water pollution,
different from the Central Pollution and
Control Board (CPCB)? [2018] (ii) Improve the quality of air and to prevent,
1) The NGT has been established by an Act control or abate air pollution in the country.
whereas the CPCB has been created by an Educational Objective: to have knowledge
executive order of the Government. about the functions of various environmental
2) The NGT provides environmental justice and organisations in India.
helps reduce the burden of litigation in the
higher courts whereas the CPCB promotes Style: How based
cleanliness of streams and wells, and aims Substance: Environmental Laws
to improve the quality of air in the country.
Form: Two statement question
Which of the statements given above is/are
correct?

a. 1 only
b. 2 only

www.laexias.com Page 75 https://elearn.laex.in


 
     
RRP 2022 - Environment UPSC
Previous Year Questions
               
46. Which of the following are the Answer • Regulation of activities aimed at prevention,
features of ‘National Ganga River Basin control and abatement of pollution in Ganga
Authority’ (NGRBA)? [2016] to maintain its water quality
1) River basin is the unit of planning and • Maintenance of minimum ecological flows in
management. the river Ganga
2) It spearheads the river conservation efforts Educational Objective: to learn about various
at the national level. initiatives towards the cleaning of River Ganga.
3) One of the Chief Ministers of the States
Style: Which based
through which the Ganga flows becomes the
Chairman of NGRBA on rotation basis. Substance: Environmental Initiatives and
Select the correct answer using the code policies
given below. Form: Three statement question
a. 1 and 2 only 4.2.1.2. Environment &
b. 2 and 3 only Wildlife Conservation Agencies
c. 1 and 3 only
d. 1, 2 and 3 47. ‘Gadgil Committee Report’ and
‘Kasturirangan Committee Report’,
Answer: A
sometimes seen in the news, are related
Explanation: to [2016]

‘National Ganga River Basin Authority’ a. Constitutional reforms


(NGRBA) b. Ganga Action Plan
c. Linking of rivers
Constituted under Environment (Protection) Act, d. Protection of Western Ghats
1986. The Ministry of Water Resources, River
Answer: D
Development and Ganga Rejuvenation (MoWR,
RD & GR) is the nodal Ministry for the NGRBA Explanation: Both the committees are related to
protection of western ghats by declaring
Chaired by the Prime Minister and has as its
Ecologically Sensitive Area.
members the Union Ministers concerned, the
Chief Ministers of the States through which Gadgil committee:
Ganga flows
The Ministry of Environment and Forests,
Functions Government of India, constituted, by an order
dated 2010, a Western Ghats Ecology Expert
• It is a planning, financing, monitoring and
Panel (WGEEP) headed by Prof. Madhav Gadgil
coordinating body of the centre and the
to assess the status of ecology, demarcate areas
states
within the Western Ghats Region for notifying
• Development of a river basin management
ecologically sensitive zones under the
plan
Environment (Protection) Act, 1986.

www.laexias.com Page 76 https://elearn.laex.in


 
     
RRP 2022 - Environment UPSC
Previous Year Questions
               
In August 2012, a High-Level Working Group on conservation organisations, with over 120
Western Ghats under Kasturirangan was partner organizations. Together the Birdlife
constituted to “examine” the Gadgil Committee Partnership forms the leading authority on the
report in a “holistic and multidisciplinary status of birds, their habitats & the issues and
fashion in the light of responses received” from problems affecting bird life.
various stakeholders. Bird Life’s Important Bird and Biodiversity Area
(IBA) Programme aims to identify, monitor and
Educational Objective: to have idea about
protect a global network of IBAs for the
various committees that are setup to protect the
conservation of the world’s birds and other
environment
wildlife.
Style: Current affairs
Educational objective: to know about the
Substance: Environmental Initiatives international environmental organizations.

Form: Single answered question Style: Reference based

48. With reference to an organization known Substance: International Environmental


as 'Birdlife International', which of the initiatives
following statements is/are correct? [2015]
Form: Three statement
1) It is a Global Partnership of Conservation
Organizations. 49. With reference to Bombay Natural
2) The concept of 'biodiversity hotspots' History Society (BNHS), consider the
originated from this organization. following statements: [2014]
3) It identifies the sites known/ referred to as 1) It is an autonomous organization under the
'Important Bird and Biodiversity Areas'. Ministry of Environment and Forests.
Select the correct answer using the code 2) It strives to conserve nature through action-
given below. based research, education and public
awareness.
a. 1 only 3) It organizes and conducts nature trails and
b. 2 and 3 only camps for the general public.
c. 1 and 3 only
Which of the statements given above is/are
d. 1, 2 and 3
correct?
Answer: C
a. 1 and 3 only
Explanation: b. 2 only
c. 2 and 3 only
Bird Life international is a global partnership
d. 1,2 and 3
of conservation organisations that strives to
conserve birds, their habitats and global
biodiversity, working with people towards
sustainability in the use of natural resources. It
is the World’s largest partnership of

www.laexias.com Page 77 https://elearn.laex.in


 
     
RRP 2022 - Environment UPSC
Previous Year Questions
               
Answer: C cannot be made without the approval of
NBA.
Explanation:
Which of the statements given above is/are
BNHS is one of the largest non-governmental correct?
organization (NGO) in India engaged in
conservation of nature through action-based a. 1 only
research, education and public awareness. Not a b. 2 and 3 only
government Institution c. 1 and 3 only
d. 1,2 and 3
Functions:
Answer: C
o It supports many research efforts through
Explanation:
grants and publishes the Journal of the
Bombay Natural History Society. National Biodiversity Authority.
• It organizes and conducts nature trails and
• Established in 2003 to implement India’s
camps for the general public
Biological Diversity Act (2002)
• Department of Science and Technology has
• Functions
designated it as a ‘Scientific and Industrial
Research Organisation’. o The Act covers conservation, use of
• BNHS is the partner of Bird Life biological resources and associated
International in India. knowledge occurring in India for
commercial or research purposes or for
Educational Objective: to know about various
the purposes of bio-survey and bio-
NGO’s that are working towards the
utilisation.
conservation of nature in India and across the
world. o It provides a framework for access to
biological resources and sharing the
Style: Reference based
benefits arising out of such access and
Substance: Environmental initiatives use.

Form: Three statement o The Act also includes in its ambit the
transfer of research results and
50. How does National Biodiversity Authority
application for intellectual property
(NBA) help in protecting the Indian
rights (IPRs) relating to Indian biological
agriculture? [2012]
resources.
1) NBA checks the biopiracy and protects the
indigenous and traditional genetic To check misappropriation (biopiracy) of Indian
resources. biological resources, the Biological Diversity Act,
2) NBA directly monitors and supervises the 2002 provides access to Indian biological
scientific research on genetic modification of resources and associated knowledge and
crop plants. obtaining the approval of the National
3) Application for Intellectual Property Rights Biodiversity Authority before seeking any IPR
related to genetic/biological resources
www.laexias.com Page 78 https://elearn.laex.in
 
     
RRP 2022 - Environment UPSC
Previous Year Questions
               
based on biological material and associated ‘Momentum for Change: Climate Neutral Now’
knowledge obtained from India is mandatory. award

The top biotech regulator in India is Genetic Style: Descriptive


Engineering Appraisal Committee (GEAC).
Substance: Global Environmental initiatives
GEAC is responsible for granting permits to
Form: Single answered
conduct experimental and large-scale open field
trials and also grant approval for commercial 52. The United Nations Framework
release of biotech crops. Convention on Climate Change (UNFCCC)
is an international treaty drawn at [2010]
Educational Objective: to have knowledge
about functions and duties of Statutory a. United Nations Conference on the Human
environmental organisations in India. Environment, Stockholm, 1972
b. UN Conference on Environment and
Style: How based Development, Rio de Janeiro, 1992
Substance: Environmental Laws and initiatives c. World Summit on Sustainable Development,
Johannesburg, 2002
Form: Three statement d. UN Climate Change Conference,
4.2.2. International Copenhagen, 2009.
4.2.2.1. Climate change Answer: B

51. "Momentum for Change: Climate Neutral Explanation: The United Nations Framework
Now" is an initiative launched by [2018] Convention on Climate Change is an
a. The Intergovernmental Panel on Climate international environmental treaty adopted on 9
Change May 1992 and opened for signature at the Earth
b. The UNEP Secretariat Summit in Rio de Janeiro from 3 to 14 June
c. The UNFCCC Secretariat 1992. It then entered into force on 21 March
d. The World Meteorological Organization 1994, after a sufficient number of countries had
ratified it. Hence option (b) is the correct answer.
Answer: C
Educational Objective: To know about
Explanation:
Important international treaties and their place
The United Nations Climate Change secretariat of signing.
has launched a new initiative that will showcase
Style: Descriptive
efforts by individuals, companies and
governments that are achieving real results in Substance: Global Environmental initiatives
transitioning to climate neutrality. Momentum
Form: Single answered
for Change: Climate Neutral. Now brings
together two of the secretariat’s flagship
activities that recognize leadership in tackling
climate change by non-Party stakeholders.
Educational Objective: Infosys won the
www.laexias.com Page 79 https://elearn.laex.in
 
     
RRP 2022 - Environment UPSC
Previous Year Questions
               
4.2.2.2. Forest
5. Sustainable
53. Bio Carbon Fund Initiative for
Sustainable Forest Landscapes' is Development
managed by the [2015]
a. Asian Development Bank 1. The Partnership for Action on Green
b. International Monetary Fund Economy (PAGE),a UN mechanism to
c. United Nations Environment Programme assist countries transition towards
d. World Bank greener and more inclusive economies,
emerged at [2018]
Answer: D
a. The Earth Summit on Sustainable
Explanation: The Bio Carbon Fund Initiative for
Development 2002, Johannesburg
Sustainable Forest Landscapes (ISFL) is a
b. The United Nations Conference on
multilateral fund, supported by donor
Sustainable Development 2012, Rio de
governments and managed by the World Bank.
Janeiro
The Bio Carbon Fund Initiative for Sustainable
c. The United Nations Framework Convention
Forest Landscapes collaborates with countries
on Climate Change 2015, Paris
around the world to reduce emissions from the
d. The World Sustainable Development
land sector through smarter land use planning,
Summit 2016, New Delhi
policies, and practices.
Answer: B
Educational Objective: To know about different
types of funds associated in saving the Explanation:
environment. The Partnership for Action on Green
Style: Descriptive Economy (PAGE), a UN mechanism to assist
countries transition towards greener and more
Substance: Global Environmental initiatives inclusive economies and to deliver
a more coordinated and effective response to
Form: Single answered
member state initiatives, emerged at the United
Nations Conference on Sustainable Development
2012, Rio de Janeiro.

Educational objective: to know about the


international organizations, their environmental
initiatives and their work towards conservation.

Style: Descriptive

Substance: International Initiatives

Form: Single answered

www.laexias.com Page 80 https://elearn.laex.in


 
     
RRP 2022 - Environment UPSC
Previous Year Questions
               
2. With reference to 'Agenda 21', sometimes Which of the statements given above is / are
seen in the news, consider the following correct?
statements: [2016]
a. 1 Only
1. It is a global action plan for sustainable b. 2 only
development. c. Both 1 and 2 only
2. It originated in the World Summit on d. Neither 1 nor 2
Sustainable Development held in
Answer: B
Johannesburg in 2002.
Which of the statements given above is / are Explanation:
correct? Club of Rome talked about “limits to growth. The
a. 1 only Sustainable Development Goals were first
b. 2 only proposed in 2012 Rio plus 20.
c. Both 1 and 2 Agenda: “Transforming Our World: the 2030
d. Neither 1 nor 2 agenda for Sustainable Development”. Therefore,
Answer: A the SDG’s have to be achieved by 2030

Explanation: Educational objective: to know about the


international environmental initiatives and their
Agenda 21 is a non-binding, voluntarily
work towards conservation.
implemented action plan of the United Nations
with regard to sustainable development. Style: Consider based

It is a product of the Earth Summit (UN Substance: International Initiatives


Conference on Environment and Development)
Form: Two statement
held in Rio de Janeiro, Brazil, in 1992.
6.2.1. Services of Ecosystem
Educational objective: to know about the
international organizations. 4. The Millennium Ecosystem Assessment
describes the following major categories
Style: Reference based
of ecosystem services- provisioning,
Substance: International Initiatives supporting, regulating, preserving and
cultural. Which one of the following is
Form: Two statement supporting service? [2012]
3. Consider the following statements: [2016] a. Production of food and water
1. The Sustainable Development Goals were b. Control of climate and disease
first proposed in 1972 by a global think c. Nutrient cycling and crop pollination
tank called the Club of Rome'. d. Maintenance of diversity
2. The Sustainable Development Goals have to Answer: C
be achieved by 2030.
Explanation: Supporting services are the
services which are responsible for production of

www.laexias.com Page 81 https://elearn.laex.in


 
     
RRP 2022 - Environment UPSC
Previous Year Questions
               
all other ecosystem services, hence option c is Answer: B
correct as nutrient cycling and pollination helps
Explanation:
in production of food which is a provisioning
service of ecosystem.

• Control of climate and disease is a


regulating service as it regulates the
ecosystem processes.
• Maintenance of diversity is a preserving and
cultural service as it increases species
richness and in turn creates recreational
value.
• Educational Objective: Nutrient cycling
and pollination helps in production of food Primary producers are the organisms who
which in turn provides services to the synthesis their own food using photosynthesis or
ecosystem. chemosynthesis.
• Form: Single statement based
• Style: Which based questions Option 1 is incorrect. Copepods are a group of
• Substance: Habitat/Protected areas-based small crustaceans found in nearly every
questions freshwater and saltwater habitat. They are
6.2.2. Functions of Ecosystem major primary consumers in the World Ocean.
Copepods are generally herbivores, feeding only
6.2.2.1. Energy flow through Food Chain
on plant plankton which they filter from the
5. Consider the following kinds of water.
organisms: (2021)
Option 2 is correct. Cyanobacteria, also
1. Copepods called blue-green algae, are microscopic
2. Cyanobacteria organisms found naturally in all types of water.
3. Diatoms These organisms are primary producers who use
4. Foraminifera sunlight to make their own food.
Which of the above are primary producers in
Option 3 is correct. Diatoms are
the food chains of oceans?
photosynthesising algae. They are found in
(a) 1 and 2 almost every aquatic environment including
(b) 2 and 3 fresh and marine waters. They are primary
(c) 3 and 4 producer in oceanic food chain.
(d) 1 and 4
Option 4 is incorrect. Foraminifera are single-
celled organisms that are found in most marine
environments. They are characterized by
streaming granular ectoplasm for catching food.
Foraminifera are heterotrophic organisms which

www.laexias.com Page 82 https://elearn.laex.in


 
     
RRP 2022 - Environment UPSC
Previous Year Questions
               
consumes smaller organisms and organic • Educational Objective: A food chain
matter. illustrates the order in which a chain of
organisms feed upon each other, Food
Educational objective: Cyanobacteria and
chains are found in of different species.
Diatoms are producers in food chain.
• Form: Three statement Based question
Form: List based question • Style: Reference Based
• Substance: Concept
Style: Consider based question
7. With reference to the food chains in
Substance: Species based question ecosystems, which of the following kinds
of organism is/are known as decomposer
6. With reference to food chains in
organism/organisms? [2013]
ecosystems, consider the following
statements: [2013] 1) Virus
2) Fungi
1) A food chain illustrates the order in which a
3) Bacteria
chain of organisms feed upon each other
2) Food chains are found within the Select the correct answer using the codes
populations of a species given below.
3) A food chain illustrates the number of each
a. 1 only
organism which are eaten by others
b. 2 and 3 only
Which of the statements given above is/are c. 1 and 3 only
correct? d. 1, 2 and 3

a. l only Answer: B
b. 1 and 2 only
Explanation: Viruses are biological agents that
c. 1,2 and 3
reproduces inside the living cells only and hence
d. None
they cannot exist without host and hence are
Answer: A not decomposers.

Explanation: Food chain represents the • Bacteria and fungi feed on dead
sequence of eaten and being eaten hence decomposed and the dead organic matter of
statement 1 is correct. plants and animals by secreting enzymes
outside their body on the decaying matter.
• Food chain represents the entire sequence
• They are also called as Saprotrophs
of organisms from primary producers to
/Detritus feeders.
tertiary consumers of different species. Ex:
• They help in recycling nutrients.
plant-caterpillar-lizard-snake. Hence
• Educational Objective: Bacteria and Fungi
statement 2 is incorrect.
depends on dead organism for feeding, virus
• Food chain indicates only the source and
reproduces only in living cells so can’t be
direction of flow of energy and not the
decomposer.
number of organisms eaten hence
• Form: List based question
statement 3 is incorrect.
• Style: Reference Based
www.laexias.com Page 83 https://elearn.laex.in
 
     
RRP 2022 - Environment UPSC
Previous Year Questions
               
• Substance: Species based question (a) Carbon cycle
8. A pesticide which is a chlorinated (b) Nitrogen cycle
hydrocarbon is sprayed on a food crop. (c) Phosphorus cycle
The food chain is: Food Crop-Rat- Snake- (d) Sulphur cycle
Hawk. In this food chain, the highest Answer: C
concentration of the pesticide would
Explanation:
accumulate in which one of the
following? [2010] Option c is correct. Carbon and Nitrogen cycle
a. Food crop are the gaseous cycles. Here, the main reservoir
b. Rat of nutrient is the atmosphere or the
c. Snake hydrosphere. In sulphur cycle, nutrient is
d. Hawk released by weathering of rocks, erosional runoff
and decomposition of organic matter. The
Answer: D
sulphur cycle is mostly sedimentary but two of
Explanation: Bio-magnification is the process of its compounds hydrogen sulphide (H2S) and
accumulation of pollutants as one moves from sulphur dioxide (SO2) add a gaseous component
lower trophic to higher trophic levels. to its normal sedimentary cycle.

The concentration of pollutant increases from In Phosphorus cycle, nutrients got released
one level to another (low to high). mainly by weathering of rocks since the
Phosphorus mainly occurs as a mineral in
In the given list of organisms, the food chain
phosphate rocks. It enters the cycle from erosion
sequence is Food-rat-snake-hawk
and mining activities.
Hence hawk being the tertiary consumers has
Educational objective: In the case of
the highest concentration of pollutant.
Phosphorus cycle biogeochemical cycles, the
Educational Objective: Bio-magnification is weathering of rocks is the main source of release
high at higher tropic levels. of nutrients to enter the cycle.

Form: Single answered based Form: Single answered based

Style: Description based question Style: Which based questions

Substance: Species based question Substance: Cause and effect-based questions

163.2.1.1. Nutrient Cycling 10. Which of the following adds/add carbon


163.2.1.1.1. Carbon Cycle dioxide to the carbon cycle on the planet
Earth? [2014]
9. In the case of which of the following
1) Volcanic action
biogeochemical cycles, the weathering of
2) Respiration
rocks is the main source of release of
3) Photosynthesis
nutrients to enter the cycle? (2021)
4) Decay of organic matter

www.laexias.com Page 84 https://elearn.laex.in


 
     
RRP 2022 - Environment UPSC
Previous Year Questions
               
Select the correct answer using the code 11. Human activities in the recent past have
given below. caused the increased concentration of
carbon dioxide in the atmosphere, but a
a. 1 and 3 only
lot of it does not remain in the lower
b. 2 only
atmosphere because of [2011]
c. 1, 2 and 4 only
d. 1, 2, 3 and 4 1) Its escape into the outer stratosphere.
2) The photosynthesis by phytoplankton in the
Key: C
oceans
Explanation: Volcanic action is the eruption of 3) The trapping of air in the polar ice caps.
interior of the earth in the form of magma during Which of the statements given above is/ are
such various gases like SO2, Carbon-di-oxide correct?
are released into atmosphere.
a. 1 and 2
• Respiration is the process of breathing by b. 2 only
living organisms which includes release of c. 2 and 3
carbon-di-oxide and intake of oxygen for d. 3 only
their metabolic activities.
Answer: B
• Photosynthesis is the ultimate producer of
food for all living organisms by the primary Explanation: Statement 1 is incorrect because
producers which produce glucose by carbon-di-oxide is found only upto 90 km from
consuming carbon-di-oxide and releasing earth surface and does not extend to outer
oxygen in the process. Therefore, Option 3 stratosphere.
can be eliminated from given choices.
• Phyto planktons are the primary producers
• Decaying of organic matter release co2
of marine ecosystem and they absorb great
which is stored as biomass during the life of
amount of co2 for their photosynthesis from
the organism.
the lower atmosphere hence co2 in lower
• Thus option 1, 2 and 4 are process
atmosphere is readily absorbed by them
responsible for releasing co2 back into
decreasing their quantity and acting as
carbon cycle.
lungs. Hence statement 2 is correct
• Education Objectives: Decaying of organic
• Trapping of air in polar ice caps contain
matter release CO2 which is stored as
large amount of co2 of historic emissions
biomass, through other processes co2 is
and not recent ones hence statement 3 is
released into the atmosphere.
incorrect.
• Form: List based question
• Educational Objective: The photosynthesis
• Style: Which based questions
by phytoplankton in the oceans absorb CO2
• Substance: Cause and effect-based
of lower atmosphere.
questions
• Form: Three statement Based question
• Style: Description based question
• Substance: Cause and effect-based
questions

www.laexias.com Page 85 https://elearn.laex.in


 
     
RRP 2022 - Environment UPSC
Previous Year Questions
               
163.2.1.2. Ecological Succession grasslands tress do not replace grasses because
of low moisture and high temperatures which
12. Lichens, which are capable of initiating
results in frequent fires in grasslands.
ecological succession even on a bare
rock, are actually a symbiotic association Insects and fungi also cannot survive in
of [2014] extremes of grasslands.
a. Algae and bacteria Hence option 3 is the correct answer.
b. Algae and fungi
c. Bacteria and fungi Educational Objective: Grass land witness
d. Fungi and mosses frequent fires and scarcity of water so trees can
not replace grass.
Answer: B
Form: Single answered based
Explanation: Lichens represents a group of
greyish green plants which grow on rocks, tree Style: Description based question
trunks, dead wood etc. It represents a symbiotic
Substance: Species based question
relation between algae and fungi.
163.2.2. Types of Ecosystem
• In this unique relationship algae
163.2.2.1. Terrestrial Ecosystem
manufactures food for fungi and fungi in
turn absorbs and retains water which keeps 14. Which of the following leaf modifications
algal cells moist. occurs/occur in desert areas to inhibit
• They are most common in wetlands, rare in water loss? [2013]
streams and rivers and absent in ground
1) Hard and waxy leaves
water.
2) Tiny leaves or no leaves
• Educational Objective: Green plants which
3) Thorns instead of leaves
grow on rocks are lichens it represents a
symbiotic relation between algae and fungi. Select the correct answer using the codes
• Form: Single answered based given below.
• Style: Description based question a. 1 and 2 only
• Substance: Species based question b. 2 only
13. In the grasslands, trees do not replace c. 1 and 3 only
the grasses as a part of an ecological d. 1, 2 and 3
succession because of [2013] Answer: D
a. Insects and fungi
Explanation: The plants in desert survive the
b. Limited sunlight and paucity of nutrients
extreme climates by xerophytic adaptations
c. Water limits and fire
which includes various modification in roots,
d. None of the above
stem and leaves. Modification of leaves is the
Answer: C frequent adaptation
Explanation: Grasslands represent the ecotone
between the tropical forests and deserts. In

www.laexias.com Page 86 https://elearn.laex.in


 
     
RRP 2022 - Environment UPSC
Previous Year Questions
               
• Hard and waxy leaves prevent excess Lake comes third as its productivity various on
transpiration and also protects leaves from level of nutrients with eutrophic being more
direct heat of sun productive. Oceans are least among the four in
• Tiny leaves and thorny leaves also regulate spite of being large in area because only up to
excess transpiration and also in turn 250 m of ocean is productive where sunlight is
protects plants from cattle. available called photic zone.
• Hence all are helpful for survival of plants in
Hence option c is the right sequence
desert climate
• Educational Objective: Hard and waxy Educational Objective: Correct sequence of
leaves, Tiny leaves or no leaves and Thorns ecosystems in the order of decreasing
instead of leaves following leaf modifications productivity is Mangroves, grasslands, lakes,
occurs/occur in desert areas to inhibit oceans.
water loss.
Form: Single answered based
• Form: List based question
• Style: Which based questions Style: Which based questions
• Substance: Cause and effect-based
questions Substance: Habitat/Protected areas-based
questions
15. hich one of the following is the correct
163.2.2.2. Aquatic Ecosystem
sequence of ecosystems in the order of
decreasing productivity? [2013] 16. Consider the following pairs [2014]
a. Oceans, lakes, grasslands, mangroves Wetlands Confluence of rivers
b. Mangroves, oceans, grasslands, lakes
c. Mangroves, grasslands, lakes, oceans 1. Harike Wetlands : Confluence of Beas
d. Oceans, mangroves, lakes, grasslands
and Satluj/Sutlej
Answer: C
2. Keoladeo Ghana : Confluence of
Explanation: Productivity is the production of National Park
new biomass by an ecosystem. Banas and Chambal

Different ecosystem has different rate of 3. Kolleru Lake : Confluence of Musi


productivity which depend on various factors and Krishna
like water, nutrients, temperature etc.
Which of the above pairs is / are correctly
In the given ecosystems, mangrove have highest matched?
productivity due to it having high species
a. 1 only
richness as it represents ecotone between
b. 2 and 3 only
terrestrial and marine ecosystem. Then comes
c. 1 and 3 only
the grasslands which represents ecotone
d. 1, 2 and 3
between tropics and desert but less productive
than mangroves due to low moisture content. Answer: A

www.laexias.com Page 87 https://elearn.laex.in


 
     
RRP 2022 - Environment UPSC
Previous Year Questions
               
Explanation: Harike wetland also called “Hari- Key: A
ke-Pattan” is the largest wetland in north India
Explanation: Wetlands are the areas saturated
constructed at the confluence of Beas and Sutlej
with water throughout the year or in certain
just south of Harike village in Punjab.
times of the year. They are ecotone of deep water
• Keoladeo Ghana National Park is a and terrestrial habitats.
manmade wetland constructed at the
• Around 4.7% of total geographical area
confluence of rivers Gambhir and Banganga
(15.3 million hectares) is covered by
in Rajasthan.
Wetlands in India.
• Kolleru lake is one of the largest fresh water
• Out of this, area under inland wetlands is
lakes in India and largest shallow water lake
69%, coastal wetlands is 27% and other
between deltas of Krishna and Godavari.
wetlands is 4%. Therefore statement 2 is
Hence only the first option is correctly matched. incorrect.
Educational Objective: Harike wetland • Among states, Gujarat has largest wetland
constructed at the confluence of Beas and Sutlej area (3 million hectares) followed by Andhra
just south of Harike village in Punjab, Kolleru Pradesh, Uttar Pradesh, west Bengal and
lake located between deltas of Krishna and Maharashtra. Therefore statement 1 is
Godavari. correct.
• Educational Objective: Out of this, area
Form: Matching based question under inland wetlands is 69% and Among
states, Gujarat has largest wetland area (3
Style: Consider based question
million hectares) followed by Andhra
Substance: Habitat/Protected areas-based Pradesh, Uttar Pradesh, west Bengal and
questions Maharashtra.
• Form: Two statements based
17. With reference to the wetlands of India,
• Style: Reference Based
consider the following statements: [2012]
• Substance: Habitat/Protected areas-based
1) The country's total geographical area under questions
the category of wetlands is recorded more in
18. What would happen if phytoplankton of
Gujarat as compared to other States.
an ocean is completely destroyed for
2) In India, the total geographical area of
some reason? [2012]
coastal wetlands is larger than that of
inland wetlands. 1) The ocean as a carbon sink would be
adversely affected.
Which of the statements given above is/are
2) The food chains in the ocean would be
correct?
adversely affected.
a. 1 only 3) The density of ocean water would drastically
b. 2 only decrease.
c. Both 1 and 2
d. Neither 1 nor 2

www.laexias.com Page 88 https://elearn.laex.in


 
     
RRP 2022 - Environment UPSC
Previous Year Questions
               
Select the correct answer using the codes Select the correct answer from the codes
given below: given below:

a. 1 and 2 only a. 1 only


b. 2 only b. 1 and 2 only
c. 3 only c. 2 and 3 only
d. 1, 2 and 3 d. 1, 2 and 3
Answer: A Answer: D

Explanation: Phyto-planktons are the primary Explanation: Algal blooms are the excessive
producers of the ocean ecosystem and form the growth of microscopic algae due to addition of
base of food chain. nutrients which as a catalyst for multiplying
algae. So, discharge of nutrients from estuaries
• Phyto-planktons being the primary
helps the algae (whether harmful or harmless) to
producers absorb great quantity of carbon-
grow.
di-oxide from the atmosphere for
photosynthesis, thus acting as carbon sink. • Run off from the land during the monsoon
• Thus, complete destruction of bring salts which favours the growth of
Phytoplankton’s affects the ocean as carbon algae. Hence statement 2 is correct.
sink and also food chain will be affected • Upwelling of seas bring the nutrients to the
destroying the entire ocean ecosystem. surface increasing the fast multiplication of
Hence statement 1 and 2 are correct. algae. Hence statement 3 is also correct.
• Statement 3 is wrong as density of ocean • Education Objective: Discharge of
water depends on the quantum of dissolved nutrients from the estuaries, Run-off from
salts and rate of evaporation. the land during the monsoon and Upwelling
• Education Objective: if phytoplankton of in the seas could lead to increase in harmful
an ocean is completely destroyed the ocean algal blooms.
as a carbon sink and the food chains in the • Form: Three statement Based question
ocean would be adversely affected. • Style: What based questions
• Form: Three statement Based question • Substance: Cause and effect-based
• Style: What based questions questions
• Substance: Cause and effect-based
questions
19. There is a concern over the increase in
harmful algal blooms in the seawaters of
India. What could be the causative
factors for this phenomenon? [2011]
1) Discharge of nutrients from the estuaries.
2) Run-off from the land during the monsoon.
3) Upwelling in the seas.

www.laexias.com Page 89 https://elearn.laex.in


 
     
RRP 2022 - Environment UPSC
Previous Year Questions
               

www.laexias.com Page 90 https://elearn.laex.in


 

You might also like